Archive for the ‘Bible historicity’ Category

h1

The Date of the Exodus (2/2)

February 19, 2015

This article is the second of two in a consideration of the date of the Exodus. Typically, only two dates are considered viable; c.1440 BCE (the ‘early date’), and c.1280 BCE (the ‘late date’). Arguments for these dates are reviewed and compared here.

Summary of Key Arguments

Here are the key arguments for the early date (c. 1440 BCE), together with criticisms.

Argument: The 480 years of 1 Kings 1:6 indicates an early date

  • This is only a relative chronology, and the number does not agree with the years recorded in the Judges[1] [2]
  • The number may be symbolic for 12 generations[3]
  • This disagrees with the date for Abraham[4]

Argument: Some destruction layers in Canaan support an early date[5]

  • Destruction and occupation layers provide more support for a late date[6]
  • No evidence for Edom and Moab existing at an early date[7]

Argument: Reference to the Habiru in the 14th century Armana letters[8]

  • Extensive study has revealed no direct correspondence between the Habiru and the Hebrews, thus the Armana letters do not support the early date[9] [10]
  • Armarna correspondence contradicts early date destruction of Hazor[11]

Argument: No evidence for 13th century occupation or destruction of Jericho, Ai, or Hazor[12]

  • Hazor was occupied and destroyed in the 13th century,[13] reliable excavation of 13th century Jericho is challenged by extensive erosion,[14] and the archaeological data for Ai is difficult to reconcile with both early and late dates;[15] these are insignificant archaeological challenges for the late date

Here are the key arguments for the late date (c. 1280 BCE), together with criticisms.

Argument: Destruction of Hazor in 13th century[16]

  • The archaeological evidence may indicate the destruction in Judges 4:24[17]

Argument: Pithom and Rameses in Exodus 1:11 are evidence for events under Pharaoh Rameses II (1279-1213 BCE)[18] [19]

  • This requires Rameses to be built before Rameses II even began to rule[20]
  • Unlike the pharaoh of the Exodus, Rameses II did not die in the Red Sea[21]
  • The sites were built earlier; one was later renamed ‘Rameses’, and an editor of the book of Exodus updated the text with this name[22]

Argument: Covenant formulas in the Law of Moses closely match those from 1400-1200 BCE[23]

  • There is an insufficient match to date the Biblical covenants precisely[24]

Argument: The Merneptah Stele (13th century), is the earliest reference to Israel in Canaan[25]

  • Egyptologist Manfred Görg has suggested an Egyptian inscription he dates to the 13th century, contains a reference to Israel which may have been copied from an 18th Dynasty record (16th-13th centuries BCE), implying Israel was in Canaan before the 13th century[26]

Argument: Egypt occupied Canaan until the 12th century[27]

  • This remains unaddressed by key proponents of the early date;[28] this contradicts completely a late date for the Exodus

Argument: A very large population entered Canaan in the late 13th century[29]

  • This remains unaddressed by key proponents of the early date; [30] there is no evidence for such a population entering Canaan in the 15th century

Review of Arguments

The early date is highly vulnerable to a range of criticisms, and has the least archaeological support. In particular, the occupation and control of Canaan by Egypt until the end of the 13th century, the lack of any evidence for a new population entering Canaan in the 15th century, the Armana correspondence, the non-existence of Edom and Moab in the 15th century, and the evidence for destruction of Canaanite sites matching a 13th century conquest rather than a 15th century conquest, are  formidable challenges to the traditional late date.

Objects to the late date are less substantial. There is no evidence that Rameses as a place name in Exodus 1 is a later editorial gloss.[31] Görg’s suggestion of an Egyptian reference to Israel earlier than the Merneptah Stele is problematic. [32] [33] Wood’s attribution of the 13th century Hazor destruction level to Deborah and Barak fails to provide evidence.[34] [35] His dating of Pithom and Rameses on the basis of the birth of Moses being described later in Exodus 1, assumes an unnecessarily strict chronological sequence for the narrative.

The pharaoh under whom Pithom and Rameses were built died while Moses was in the wilderness before the Exodus[36] (matching Rameses II and a late date Exodus). Additionally, Hoffmeier argues Exodus does not represent the pharaoh of the Exodus as dying in the Red Sea,[37] whereas an early date pharaoh would have to be Thutmose III or Amenhotep II, neither of whom died by drowning.[38]

Conclusion

Although vigorous debate over date of the Exodus is ongoing, the 13th century date continues to be held widely among those scholars who accept the historicity of the Exodus.[39] As early as 1999 Hoffmeier observed ‘Dating the period of the oppression and exodus to the fifteenth century B.C. has largely been replaced in favor of a thirteenth-century date’.[40]


[1] The years of the judges, if added sequentially, result in 633-650 years between the exodus and the reign of Solomon, compelling supporters of the early date to read the years in Judges less literally, in order to read the years in 1 Kings 6:1 more literally; ‘To get around the dilemma caused by the difference between 480 and 633-650 years, advocates of the 15th-century (and the later date) exodus date are forced to harmonize the conflicting data by proposing some overlap between judgeships to bring the 480-year figure into alignment with the 633–650 year total.12  By doing this, one abandons a straightforward, literal reading of the Judges through Exodus narratives.’, Hoffmeier, ‘What is the Biblical Date For the Exodus? A Response to Bryant Wood’, Journal of the Evangelical Theological Society, 50/2 (2007), 228.

[2] ‘When one seeks to reconstruct the numbers given in the biblical accounts, consistently and literally, they do not add up to the number 480 given in 1 Kgs 6:1.’, Hawkins, ‘Propositions For Evangelical Acceptance Of A Late-Date Exodus-Conquest:  Biblical Data And The Royal Scarabs From Mt. Ebal’, Journal of the Evangelical Theological Society, 50/1 (2007), 35.

[3] ‘It has long been thought that the 480-year figure of 1 Kgs 6:1 might be a symbolic figure that derives from 12 times 40-40 years being a symbolic number for a generation—thus signifying that 12 generations had elapsed between the exodus and Solomon’s 4th year. Since men were usually married and had children by age 20–25, 60  a period closer to 300 years would be more accurate. When one adds 300 to 967 BC, an Exodus date around 1267 BC (20 years into the reign of Ramesses II) results.’, ibid., p. 236.

[4] ‘A 15th-century B.C. date presents problems for the chronology of Abram. Archaeological evidence relating to the overthrow of Sodom and Gomorrah seems to date Abram’s arrival in Canaan around 1900 B.C. The Genesis narratives place Jacob’s migration to Egypt about 215 years later. On the basis of the 430 years of Exodus 12:40 it would seem that Abram came to Canaan about 2086 B.C., some 645 years before the exodus. That would date his birth (cf. 12:4) about 2161 B.C. If the Sodom and Gomorrah evidence is correct, Abram’s arrival in Canaan would harmonize with a 13th-century B.C. date.’, Harrison, ‘Exodus, The’, in Elwell & Beitzel, Baker Encyclopedia of the Bible (1988), 743-744.

[5]According to Wood, some archaeological findings—such as destruction layers from Jericho, Ai and Hazor—support a 15th-century exodus (Wood, “The Rise and Fall of the 13th-Century Exodus-Conquest Theory,” 488–89; Wood, “From Ramesses to Shiloh,” 256–82).’, Thornhill, ‘Exodus’, in Barry & Wentz (eds.), The Lexham Bible Dictionary (2012).

[6] ‘The destruction and occupation layers of many conquest cities (e.g., Lachish, Debir, Hazor, Bethel, etc.) favor the 13th-century dating.’, ibid.

[7] ‘Excavation findings seem to indicate that Edom and Moab (compare Exod 13:15; Num 20:14–21) were not yet established peoples during the mid-14th century.’, ibid.

[8] ‘Wood also cites the mention of the ‘ (‘)apiru in the Canaanite Amarna letters of the mid-14th century, as well as an inscription dating to the 18th Dynasty. This inscription appears to mention Ashkelon, Canaan, and Israel (Wood, “The Rise and Fall of the 13th-Century Exodus-Conquest Theory,” 489).’, Thornhill, ‘Exodus’, in Barry & Wentz (eds.), The Lexham Bible Dictionary (2012).

[9] ‘The relationship between the “Habiru” of the Amarna letters, the “Apiru” in 13th-century B.C. Egypt, and the biblical Hebrews has been examined minutely by scholars. Widely differing opinions have been offered. Some believe that the three are variations of the name of one people. To others, however, it seems far from clear that there was any significant relationship between the names. Such disagreement also tends to intensify the problem.’, Harrison, ‘Exodus, The’, in Elwell & Beitzel, Baker Encyclopedia of the Bible (1988), 744.

[10] ‘The ʿapiru (sometimes ḫapiru or ḫabiru) are considered to be warlords, brigands and disenfranchised peoples on the outskirts of society. Rainey has demonstrated that the term cannot be etymologically related to “Hebrew,” and the range of use of the term makes it clear that the ʿapiru cannot be equated with Israelites. Nevertheless, some would contend that it does not entirely rule out the possibility that Israelites, along with other peoples, could have been designated by the term.’, Walton, ‘Exodus, Date of’, in Alexander & Baker, Dictionary of the Old Testament: Pentateuch (2003), 263.

[11]According to the Wood, the marauding Habiru of the Amarna Letters could be the Hebrews.113  Abi-Milku, however, makes clear that Hazor was an ally of the Habiru rather than being the destroyers of Hazor. This information from the Amarna correspondences demonstrates that Hazor during the LB IIA was a major player in the region and does not sound like a city that had just been demolished and burnt by Joshua and his forces.’, Hoffmeier, ‘What is the Biblical Date For the Exodus? A Response to Bryant Wood’, Journal of the Evangelical Theological Society, 50/2 (2007), 245.

[12] ‘Only three cities are recorded as having been destroyed by fire by the Israelites: Jericho (Josh 6:24); Ai (Josh 8:28); and Hazor (Josh 11:11).14  All three pose problems for a late 13th-century conquest. At Jericho and Ai, no evidence has been found for occupation in the late 13th century, let alone for a destruction at that time.’, Wood, ‘The Rise and Fall of the 13th-Century Exodus-Conquest Theory’, Journal of the Evangelical Theological Society, 48/3 (2005), 477.

[13] ‘In Canaan, the drastic destruction of Hazor (level 13) in the later 13th century B.C. (despite misconceptions to the contrary) may well reflect Joshua’s exploit.’,  Kitchen, ‘Exodus, The,’, in Freedman (ed.), Anchor Yale Bible Dictionary (1992), 702.

[14] ‘at Jericho, nearly half a millennium of erosion has long since removed virtually all pertinent evidence.’, ibid., p. 702.

[15] ‘Ai remains an enigma on any view;’, ibid., p. 702.

[16] ‘In Canaan, the drastic destruction of Hazor (level 13) in the later 13th century B.C. (despite misconceptions to the contrary) may well reflect Joshua’s exploit.’,  Kitchen, ‘Exodus, The,’, in Freedman (ed.), Anchor Yale Bible Dictionary (1992), 702.

[17]Following the 1230 bc destruction, there was no urban center there until the time of Solomon in the 10th century bc (1 Kgs 9:15).16  The defeat of Jabin, king of Hazor, by a coalition of Hebrew tribes under the leadership of Deborah and Barak is recorded in Judges 4–5. Judges 4:24 indicates that the Israelites destroyed Hazor at this time: “And the hand of the Israelites grew stronger and stronger against Jabin, the Canaanite king, until they destroyed him.”17  If Joshua destroyed Hazor in 1230 bc, then there would be no city for the Jabin of Judges 4 to rule.’, Wood, ‘The Rise and Fall of the 13th-Century Exodus-Conquest Theory’, Journal of the Evangelical Theological Society, 48/3 (2005), 477.

[18] ‘Egyptologists have long understood the reference to Rameses to refer to Pi-Ramesses, the delta metropolis built by Ramesses II, the 19th Dynasty monarch who reigned from 1279–1213 BC.’, Hoffmeier, ‘What is the Biblical Date For the Exodus? A Response to Bryant Wood’, Journal of the Evangelical Theological Society, 50/2 (2007), 231.

[19] ‘The archaeological data is now unequivocal: Pi-Ramesses is located at modern-day Qantir, near Faqus, and was built by Ramesses II beginning around 1270 BC;’, ibid., pp. 232-233.

[20] ‘Since Moses was 80 years of age at the time of the exodus (Exod 7:7), the building of Rameses would have taken place well before Moses’ birth in 1340 bc (according to the 13th-century theory), long before Rameses came to the throne.’, Wood, ‘The Rise and Fall of the 13th-Century Exodus-Conquest Theory’, Journal of the Evangelical Theological Society, 48/3 (2005), 478.

[21] ‘Obviously, Rameses II did not drown in the yam sup, [commonly translated ‘Red Sea’] as he died of natural causes some 47 years after the presumed exodus date of 1260 bc.’, ibid., p. 478.

[22] ‘It is clear, then, that the name Rameses used in Exod 1:11 is an editorial updating of an earlier name that went out of use.’, ibid., p. 478.

[23] ‘Scholars have understood for some time, since the work of Mendenhall, Kline, and Kitchen, that the book of Deuteronomy has the literary and legal form that characterized late second millennium BC Hittite international treaties.’,  Niehaus, ‘Covenant and Narrative, God and Time’, Journal of the Evangelical Theological Society 53/3 (2010), 550.

[24] ‘The format of the biblical material is varied and complex and cannot be dated to a particular time period based on ANE treaty documents’, Wood, ‘The Rise and Fall of the 13th-Century Exodus-Conquest Theory’, Journal of the Evangelical Theological Society, 48/3 (2005), 480-481.

[25] ‘The Merneptah stela is also cited as evidence for this date, since Israel is referenced as a people group rather than a nation.’, Thornhill, ‘Exodus’, in Barry & Wentz (eds.), The Lexham Bible Dictionary (2012).

[26] ‘Due to the similarity of these names to the names on the Merenptah stela, Gorg suggests the name list may derive from the time of Rameses II, but adopting an older name sequence from the 18th Dynasty. This evidence, if it holds up to further scrutiny, would also support a 15th-century bc exodus-conquest rather than a 13th-century bc timeframe.’, Wood, ‘The Rise and Fall of the 13th-Century Exodus-Conquest Theory’, Journal of the Evangelical Theological Society, 48/3 (2005), 489.

[27] ‘In trying to work out an evangelical understanding of the emergence of Israel, Mark Chavalas and Murray Adamthwaite have recently noted that certain conditions in the archaeology of Palestine appear to mitigate against the traditional early date positioning of the Exodus/Conquest.12  They note that, at a series of sites all over Palestine, “the clear picture is that Egyptian occupation continued until the end of the Late Bronze Age (1200 BC).”’, Hawkins, ‘Propositions For Evangelical Acceptance Of A Late-Date Exodus-Conquest:  Biblical Data And The Royal Scarabs From Mt. Ebal’, Journal of the Evangelical Theological Society, 50/1 (2007), 34.

[28] For example, it is never mentioned by Bryant Wood (foremost proponent of the early date), in his key articles ‘The Rise and Fall of the 13th-Century Exodus-Conquest Theory’, Journal of the Evangelical Theological Society, 48/3 (2005), 475-489, and ‘The Biblical Date For The Exodus Is 1446 BC:  A Response To James Hoffmeier’, Journal of the Evangelical Theological Society, 50/2 (2005), 249-258.

[29] ‘The implication seemed clear that a new population group had arrived in the Central Hill-Country during the transition from the Late Bronze Age to the Iron Age I.’, Hawkins, ‘Propositions For Evangelical Acceptance Of A Late-Date Exodus-Conquest:  Biblical Data And The Royal Scarabs From Mt. Ebal’, Journal of the Evangelical Theological Society, 50/1 (2007), 34.

[30] ‘While this material has seemed to point toward a late date for Israel’s emergence in Canaan,9  it has largely gone unnoticed by evangelical scholars writing histories of Israel10  or commentaries on Joshua.’, ibid., p. 34.

[31] ‘The toponym Rameses (רַעַמְס) occurs five times in the OT, in Gen 47:11; 53  Exod 1:11; 12:37; and Num 33:3, 5. In none of these cases is the formula “old name +הוא+ new name” used, nor does a longer explanatory gloss with the word לָרִאשֹׁנה—“at the first” occur with any of the five citations. In other words, there is no evidence within these five passages to suspect that “Rameses” is an editorial gloss.’, Hoffmeier, ‘What is the Biblical Date For the Exodus? A Response to Bryant Wood’, Journal of the Evangelical Theological Society, 50/2 (2007), 234.

[32] ‘Görg’s reading of this name as “Israel” is plagued by serious linguistic and orthographic problems that preclude it from being Israel.’, Hoffmeier, ‘What is the Biblical Date For the Exodus? A Response to Bryant Wood’, Journal of the Evangelical Theological Society, 50/2 (2007), 241.

[33] ‘Especially given the absence of Israel from the Armana evidence, this seems intrinsically unlikely, given the early date and lacking a full reading.’, Fleming, The Legacy of Israel in Judah’s Bible: History, Politics, and the Reinscribing of Tradition (2012), 241.

[34] ‘A close reading of the text indicates that God gave Israel victory over her oppressors in a major battle 25 miles away from Hazor, but the text is absolutely silent regarding any military action against Hazor itself. Furthermore, the terminology used in 4:23–24 is not found in Joshua or Judges to indicate attacks on cities. Consequently, there is no basis to believe that the destruction of the final LB IIB (late 13th century) city was caused by Deborah and Barak’s triumph over Jabin and Sisera’, Hoffmeier, ‘What is the Biblical Date For the Exodus? A Response to Bryant Wood’, Journal of the Evangelical Theological Society, 50/2 (2007), 244.

[35] ‘From the Amarna letters, written to the pharaohs Amernhotep III and Akhenaten between 1390–1340 BC, we learn that Hazor was thriving during this period.’, ibid., p. 245.

[36] Exodus 2:23 During that long period of time the king of Egypt died, and the Israelites groaned because of their slave labor. They cried out, and their desperate cry because of their slave labor went up to God.

[37] ‘Psalm 136:15 may be the closest to suggest that pharaoh drowned in the seas, but that may be due to misleading English translations, e.g. JB: “Drowned Pharaoh and his army”; NIV: “swept pharaoh and his army into the Red Sea”; KJV and NAS: “He overthrew Pharaoh. .. into the Red Sea.” The key word here is נאר, which is the word used in Exod 14:27. נאר, means to “shake off” (Ludwig Koehler and Walter Baumgartner, The Hebrew and Aramaic Lexicon of the Old Testament [Leiden: Brill, 2001] 707). Nothing in this term suggests that pharaoh drowned in the sea. In fact, there is nothing to suggest in the various texts, especially in Exodus, that pharaoh led the chariot corps in pursuit of the escaping Hebrews. Perhaps people have been influenced by Cecil B. DeMille’s portrait of angry Ramesses (Yul Brynner) leading the attack at the sea. But even in The Ten Commandments, Ramesses does not follow the Israelites into the sea!’, Hoffmeier, ‘What is the Biblical Date For the Exodus? A Response to Bryant Wood’, Journal of the Evangelical Theological Society, 50/2 (2007), 239.

[38] ‘The second problem for Wood’s exodus pharaoh drowning in the sea is that the mummy of Thutmose III was found in the Deir el-Bahri cache, while Amenhotep IIs was actually discovered in his tomb, one of only a few royal mummies discovered intact.81  In fact, all the mummies of the 15th century are accounted for.82  According to the X-rays and investigations of these mummies, none indicate a death by drowning.’, ibid., p. 240.

[39] ‘The need for discussing the latter premise is that many biblical scholars who affirm the historicity of the exodus now date it to the thirteenth century B.C., questioning concrete numbers in the Bible that taken literally would place the exodus in the fifteenth century B.C.’, Petrovich, ‘Amenhotep II And The Historicity Of The Exodus-Pharaoh’, The Master’s Seminary Journal, 17/1 (2006), 83.

[40] ‘Dating the period of the oppression and exodus to the fifteenth century B.C. has largely been replaced in favor of a thirteenth-century date, although a few adherents to the earlier date have followed Jack’s thesis.’, Hoffmeier, Israel In Egypt: The Evidence for the Authenticity of the Exodus Tradition (1999), 125.

h1

The Date of the Exodus (1/2)

February 19, 2015

Despite over a century of detailed investigation, the date of the Hebrew exodus from Egypt remains a topic of extensive debate within scholarship.[1] Scholarly discussion focuses on exegetical concerns such as the interpretation of chronological data in the Old Testament, the correct identification of toponyms (place names), and the relationship between textual and archaeological data. This initial article summarizes the emergence of the two most commonly proposed dates.

History of Interpretation

Throughout the 19th century, Rameses II was considered the pharaoh under whom the Hebrews were enslaved, and his son Merneptah the pharaoh of the exodus.[2]  However, discovery of the Merneptah Stele, referring to Israel as a recognized people settled in Canaan by the 14th century, invalidated this view.

‘This new data appeared to require that Israel had already been settled there by the end of the 13th century BC. Placing Israel in Canaan this early in the reign of Merneptah raised obstacles for his having been the pharaoh of the Exodus. Israel obviously could not have left Egypt in the first year of Merneptah’s reign, wandered in the wilderness for forty years, and then appeared in Canaan as a settled ethnic group in his fifth year.’[3]

A revised interpretation identified Ramases II as the pharaoh of the exodus, a view which remained dominant throughout the rest of the 19th century, up to the 1920s.[4] [5] In 1925 a 15th century date was proposed.

‘This approach seems to have been pioneered initially by James Jack, who challenged the 13th-century BC date in his 1925 book, The Date of the Exodus in the Light of External Evidence. Jack argued that both biblical and extrabiblical evidence pointed to a mid-15th century BC date.’[6]

Textual Evidence

The early date rests principally on an application of the chronology given in 1 Kings 6:1, which appears to date the exodus 480 years before the reign of Solomon.[7] Since there is considerable agreement that Solomon’s reign started at around 960-960 BCE, counting 480 years back from this date places the exodus at c.1440 BCE. A second text seen as corroborating the early date is Judges 11:26, in which Jephthah’s claim that Israel had already been in Canaan for at least three hundred years[8] would appear to suggest an early date for the exodus.[9]

The late date receives textual support from Exodus 1:11, which refers specifically to the Hebrews building Pithom and Rameses for the Pharaoh.[10] Unlike 1 Kings 6:1 and Judges 11:26, this text rests on an absolute rather than a relative date; the construction of buildings at Pithom and Rameses.

‘Late-date theorists argue that, since the Exodus account used the name by which the city was known for about two centuries only (c. 1300–1100 B.C.), the Hebrew tradition of the exodus must also date from that period. In such an event Rameses II would have been the pharaoh of the oppression, and his son Merneptah (1224?–1214 B.C.) the pharaoh of the exodus.’[11]

Archaeological Evidence for the Conquest

In the absence of direct archaeological evidence for the Hebrew settlement in Goshen, and the lack of Egyptian records describing the Hebrews as an enslaved ethnic group, or the plagues, or subsequent exodus, attempts to date the exodus using archaeological evidence focused on dating the Hebrew entry into Canaan, searching for evidence of conquest.

Attempts have been made by proponents for both dates, and interpretation of the archaeological record has been much contested. In the 1930s, archaeologist John Garstang’s excavations of Tell es-Sultan led him to conclude there was strong evidence for a Hebrew destruction of Jericho before 1400 BCE, lending weight to an early date exodus.[12]

However, Kathleen Kenyon’s subsequent investigation of the site re-dated the destruction to around 1500 BCE, too early for the Hebrews.[13] Efforts by Bryant Wood to defend Garstang’s dating and attribute the destruction of Jericho to an early date Hebrew conquest,[14] [15] have not gained any significant scholarly acceptance outside evangelical circles. Contemporary with Garstang, archaeologist Wiliam Albright arrived at a late date for the exodus, on the basis of his investigations of Canaanite archaeological sites.[16]

In response to acknowledged difficulties harmonizing an early date with the archaeological record, in the 1970s John Bimson proposed a Hebrew conquest during the end of the Middle Bronze Age, which seemed to fit the Biblical record more closely.[17] However, Bimson’s interpretation was critiqued strongly by many scholars.

‘The critique of Bimson’s proposal came from numerous quarters. Bietak objected that his suggested alteration was only fifty years, therefore still in the sixteenth century, and could not be stretched as far as Bimson needed it to be. B. Halpern objected that the changes suggested by Bimson would leave a reduced time span for LB I that could not possibly accommodate the archaeological data.’ [18]

Archaeological evidence for destruction and occupation layers supporting a late date exodus, was considered more abundant.

‘Archaeological evidence from Canaanite sites such as Bethel, Debir, Lachish, and Hazor indicates destruction at 13th-century B.C. levels, a fact generally regarded as relating to the Hebrew occupation under Joshua.’[19]

Evidence from Philistine sites tends to favor a late rather than an early date for the exodus. Against that, however, must be set the fact that the major Philistine occupation of the southern Palestinian coastlands only occurred around 1175 B.C., in the time of Rameses III.’[20]

The Ongoing Dispute

By the 1970s the date of the exodus had ceased to become a significant concern within critical scholarship, as many commentators no longer believed in the essential historicity of the event. However, the issue continued to be debated hotly among evangelical and other faith professing scholars, as well as among a minority report of critical scholars and those professional archaeologists who considered the Biblical exodus account to preserve an essentially accurate historical core.

The second article in this series will compare and contrast the evidence and arguments advanced for each date, together with their respective counter-arguments.


[1]The date of the Exodus is one of the most debated topics in OT studies because of the ambiguous nature of the evidence.’, Shea, ‘Exodus, Date of the’, in Bromiley et al. (eds.), The International Standard Bible Encyclopedia, Revised, volume 2 (1979–1988), 230.

[2] ‘With the identification of Ramesses II as the pharaoh of the oppression, his son Merneptah, who succeeded him on the throne, naturally became the pharaoh of the Exodus. Based on this reasoning, the biblical Exodus was securely located by scholars within the 19th Dynasty of Egypt (1293–1185 BC) throughout the nineteenth century.’, Hawkins, ‘Propositions For Evangelical Acceptance Of A Late-Date Exodus-Conquest:  Biblical Data And The Royal Scarabs From Mt. Ebal’, Journal of the Evangelical Theological Society, 50/1 (2007), 31-32.

[3] Ibid., p. 32.

[4] ‘Up until about 1925, this position was widely held by scholars, both evangelical and otherwise.’, ibid., p. 32.

[5]At the beginning of the 20th century many scholars, both liberal and conservative, placed the date toward the end of the 13th century B.C.’, Harrison, ‘Exodus, The’, in Elwell & Beitzel, Baker Encyclopedia of the Bible (1988), 742.

[6] Hawkins, ‘Propositions For Evangelical Acceptance Of A Late-Date Exodus-Conquest:  Biblical Data And The Royal Scarabs From Mt. Ebal’, Journal of the Evangelical Theological Society, 50/1 (2007), 33.

[7] 1 Kings 6:1 In the four hundred and eightieth year after the Israelites left Egypt, in the fourth year of Solomon’s reign over Israel, during the month Ziv (the second month), he began building the LORD’s temple.

[8] Judges 11:26 Israel has been living in Heshbon and its nearby towns, in Aroer and its nearby towns, and in all the cities along the Arnon for three hundred years! Why did you not reclaim them during that time?

[9] ‘If 1100 BC is taken as an approximate date for Jephthah’s activities, this would place the taking of the Transjordan under Moses (Numbers 21) around 1400 BC, about 40 years after the departure from Egypt.’, Hawkins, ‘Propositions For Evangelical Acceptance Of A Late-Date Exodus-Conquest:  Biblical Data And The Royal Scarabs From Mt. Ebal’, Journal of the Evangelical Theological Society, 50/1 (2007), 33.

[10] Exodus 1:11 So they put foremen over the Israelites to oppress them with hard labor. As a result they built Pithom and Rameses as store cities for Pharaoh.

[11] Harrison, ‘Exodus, The’, in Elwell & Beitzel, Baker Encyclopedia of the Bible (1988), 743.

[12] ‘Garstang identified several levels of debris there, indicating that the city had been rebuilt a number of times. He concluded that the one built about 1500 B.C. was the Jericho overthrown by Joshua’s forces (Jos 6). Garstang’s statement that Jericho had fallen before 1400 appeared to support the time frame of 1 Kings 6:1, and was received enthusiastically by supporters of the 15th-century B.C. date.’, ibid., p. 743.

[13] ‘Garstang’s discoveries at Jericho have been modified seriously by the subsequent work of another archaeologist, Kathleen Kenyon. She found no trace of Late Bronze Age walls, which indicates that the city Garstang thought to have been conquered by Joshua was considerably earlier than his time. Unfortunately, the mound has been so ravaged by erosion and human pillaging that it reveals almost nothing about the Jericho of Joshua’s day, and thus does not help to simplify matters.’, ibid., p. 744.

[14] Wood, ‘Did the Israelites Conquer Jericho? A New Look at the Archaeological Evidence’, Biblical Archaeology Review, 16/2 (March/April 1990), 44-58.

[15] Wood, ‘The Walls of Jericho’, Bible and Spade 12/2 (1999).

[16] ‘The 13th-century exodus-conquest theory was formulated by William F. Albright in the 1930s, based largely on Palestinian archaeological evidence, and promoted by him throughout his career.’, Wood, ‘The Rise and Fall of the 13th-Century Exodus-Conquest Theory’, Journal of the Evangelical Theological Society, 48/3 (2005), 473.

[17] ‘The Late Bronze Age (LB) was characterized by a problematic lack of fortified cities. The MB in Canaan, in sharp contrast, featured the massive and numerous walled cities that the books of Numbers and Joshua seem to suggest, and many of them were destroyed at the end of the period.’, Walton, ‘Exodus, Date of’, in Alexander & Baker, Dictionary of the Old Testament: Pentateuch (2003), 259.

[18] Ibid., p. 259.

[19] Harrison, ‘Exodus, The’, in Elwell & Beitzel, Baker Encyclopedia of the Bible (1988), 744.

[20] Ibid., p. 744.

h1

Living On The Edge: challenges to faith

September 1, 2013

Today Christians in the Western world are typically living in a post-Christian society. Christian beliefs are met with skepticism, and people see little reason to believe. Christians are confronted with daily challenges to their faith, and often struggle to understand the relevance of Christianity to modern life.

The book ‘Living On The Edge: challenges to faith‘ (due to be printed in November 2013), addresses those concerns. For an overview of the book, click here.

h1

Is the Bible’s chronology of the kings of Israel accurate?

July 30, 2011

The Challenge

In the late 19th century, critical scholar Julius Wellhausen claimed the Biblical chronology of the kings of Israel was a literary invention for religious purposes, which had been edited and revised several times from a variety of different sources, rather than a genuine historical record.[1]

For the next 70 years, critical scholars continued to treat the chronology as historically worthless and irreconcilable.[2]

The Facts

In 1951, Biblical scholar Edwin Thiele published ‘The Mysterious Numbers of the Hebrew Kings’, a harmonization of the Biblical record of the kings of Israel (originally as a doctoral dissertation). By the time of the second edition (slightly revised), it was recognized that Thiele’s work was a significant breakthrough in establishing the historical validity of the Biblical chronology.[3]

Reception

Though criticisms have been made of Thiele’s chronology,[4] [5] [6] [7] its value and general validity have been acknowledged widely.[8] [9] [10] [11] [12]

It remains the typical starting point for study of the chronology of the kings of Israel and Judah[13] [14] with few modifications,[15] [16]   and has been applied successfully in other fields of Ancient Near East study, such as the chronologies of Assyria and Babylon.[17]

The reliability of the chronologies in 1-2 Kings has been supported by archaeological evidence; Grabbe notes that the chronology in these books ‘agrees with what can be gleaned from extra-biblical sources’, and that ‘even if we had no external sources we could have reasonable confidence in the biblical sequence’.[18]


[1] ‘That a process of alteration and improvement of the chronology was busily carried on in later times, we see from the added synchronisms of the kings of Israel and Judah,’, Wellhausen, ‘Prolegomena to the History of Israel’, p. 278 (1885).

[2] ‘Driver remarked that, “the length of the reigns of the various kings is not the same according to the traditional and the synchronistic figures. Since, however, it is clear on various grounds that these synchronisms are not original, any attempt to base a chronological scheme on them may be disregarded.” Kittel stated his view that, “Wellhausen has shown, by convincing reasons, that the synchronisms within the Book of Kings cannot possibly rest on ancient tradition, but are on the contrary simply the products of artificial reckoning. . . The Israelitish numbers and the parallel numbers referring to Judah do not agree at the points at which we are able to compare them.” Robinson also was impressed by Wellhausen’s evaluation: “Wellhausen is surely right in believing that the synchronisms in Kings are worthless, being merely a late compilation from the actual figures given.” * R. H. Pfeiffer’s opinion was that, “The chronology based on the synchronisms is of course less reliable than the one based on the regnal periods, since the synchronisms were figured from the regnal periods. Neither chronology is wholly accurate . . . In spite of these discrepancies, inaccuracies, and errors, the chronology of Kings is not fantastic.” 5 J. F. McCurdy expressed himself to the effect that, “Many of the numbers given, especially the synchronisms, are erroneous, as is proved by the fact that no attempt to harmonize the two series has been successful . . . Startling inconsistencies are also found where the several synchronisms for the same king are worked out.” K. Marti gave his observation: “The synchronistic notes betray their character as ‘subjective additions of the Epitomator.’ It is clear, to begin with, that this noting of synchronisms was not in actual use during the existence of the two kingdoms. . . Almost along the whole line, the discrepancy between synchronisms and years of reign is incurable.” C. H. Gordon observed: “The numerical errors in the Books of Kings have defied every attempt to ungarble them. Those errors are largely the creation of the editors who set out to write a synchronistic history of Judah and Israel, using as sources two sets of unrelated court chronicles. Combining two elaborate sets of figures was not an easy task. But even with due regard for the difficulties involved, the editors did not execute the synchronisms skillfully.”’, Thiele, ‘Synchronisms of the Hebrew Kings – A Re-evaluation: I’, Andrews University Seminary Studies, pp. 14-125 (1), 1963.

[3]A marked advance in biblical scholarship was made in the publication of The Mysterious Numbers of the Hebrew Kings, U. of Chicago Press, 1951, by Dr. E. R. Thiele. In his revised edition in 1965 (Eerdmans Pub. Co., Grand Rapids, Mich.), Dr. Thiele asserts the soundness of his basic thesis and conclusions as confirmed by scholars since his first edition.’, editorial, Journal of the Evangelical Theological Society (9.1.60), 1966.

[4] ‘Thiele’s view contains positive elements, but it also poses numerous difficulties. He incorrectly understood the annals of Tiglath-pileser III, and his determination that Menahem died in 742 contradicts the testimonies of the contemporaneous Assyrian inscriptions.10 In his desire to resolve the discrepancies between the data in the Book of Kings, Thiele was forced to make improbable suppositions. He assumed that the system of counting the years of reign changed every few generations, or even after a few decades. This is improbable, and cannot be proved. Similarly, he presumes that the Northern and Southern Kingdoms numbered their years both by the local count and by that practiced by the other kingdom, also for short periods, while this practice ceased in other periods. Thiele even went so far as to assume that while this practice had fallen into disuse, there were scribes who continued to calculate the years in accordance with it. There is no basis for Thiele’s statement that his conjectures are correct because he succeeded in reconciling most of the data in the Book of Kings, since his assumptions regarding Biblical chronological principles are derived from the chronological data themselves, whose reliability is unclear.’, Galil, ‘The Chronology of the Kings of Israel and Judah’, p. 4 (1996).

[5]but his harmonizing approach has not gone unchallenged, especially because of the many shifts in the basis of reckoning dates that it requires (e.g., Jepsen 1968: 34–35)—shifts which were unlikely in actual practice. The numerous extrabiblical synchronisms he invokes do not always reflect the latest refinements in Assyriological research (cf. E.2.f below). In many cases, he posits an undocumented event in order to save a biblical datum (e.g., the circumstances surrounding the appointment of Jeroboam II as coregent; Thiele 1983: 109)’, Cogan, ‘Chronology (Hebrew Bible)’, in Freedman, (ed.), ’The Anchor Yale Bible Dictionary’, volume 1, p. 1006 (1996)’, Freedman, DN (1996).

[6]  ‘Despite that fact of scholarly dedication, neither Thiele’s carefully argued University of Chicago dissertation, nor anyone else’s, has achieved as yet universal acceptance.’, Kaiser, ‘A History of Israel: From the bronze age through the Jewish Wars’, p. 293 (1998).

[7]Not all scholars are convinced by this solution, and commentators on the prophetic books often accept that dates can only be approximate.’, McConville, ‘Exploring the Old Testament, Volume 4: The Prophets’, p. viii (2002).

[8] ‘The chronology most widely accepted today is one based on the meticulous study by Thiele. Wiseman, ‘1 and 2 Kings’, Tyndale Old Testament Commentaries, p. 27 (1993).

[9]Increasingly his chronological scheme has come to dominate the majority of scholarly works and it is unlikely that his system can ever be overthrown without altering some well-established dates in Near Eastern history, for Thiele’s chronology is now inextricably locked into the chronology of the Near East.’, McFall, ‘A Translation Guide to the Chronological Data in Kings and Chronicles’, Bibliotheca Sacra (148.589.42-43), 1996.

[10] ‘Thiele’s system of chronology has been well received over the past 40 years and is now accepted as the basis for Israel’s chronology in a growing number of standard scholarly works.’, ibid., p. 42; see for example: Mitchell, ‘Israel and Judah until the Revolt of Jehu (931-841 B.C.)’, Cambridge Ancient History, volume 3, part 1, p. 445 (1982); Finegan, ‘Handbook of Biblical Chronology’, p. 249 (rev. ed.1998); Hess, ‘Chronology (Old Testament)’, in Porter (ed.), ‘Dictionary of Biblical Criticism and Interpretation’, p. 55 (2007).

[11]Thiele’s chronology is fast becoming the consensus view among Old Testament scholars, if it has not already reached that point.’, McFall, ‘The Chronology of Saul and David’, Journal of the Evangelical Theological Society (53.101.215), 2010.

[12]Thiele’s chronology (which differs from that of the present paper in only a few places) won the respect of historians because its dates agree with the following dates in Assyrian and Babylonian history: the Battle of Qarqar in 853 bc; the tribute of Jehu to Shalmaneser III in 841 bc; the capture of Samaria by Shalmaneser V in 723 bc; the invasion of Sennacherib in 701 bc; the Battle of Carchemish in 605 bc; and the first capture of Jerusalem by Nebuchadnezzar in 597 bc.’, Young, ‘Tables of Reign Lengths From the Hebrew Court Recorders’, Journal of the Evangelical Theological Society (48.2.232), 2005.

[13] ‘Thiele’s work has become a cornerstone of much recent chronological discussion (cf. De Vries IDB 1: 580–99; IDBSup: 161–66);’, Cogan, ‘Chronology (Hebrew Bible)’, in Freedman, (ed.), ’The Anchor Yale Bible Dictionary’, volume 1, p. 1006 (1996)’, Freedman, DN (1996).

[14] ‘Although some would prefer to see transmission errors where Thiele invokes the above principles, his chronology remains the starting point for all discussions of the debate.’, Hess, ‘Chronology (Old Testament)’, in Porter (ed.), ‘Dictionary of Biblical Criticism and Interpretation’, p. 55 (2007).

[15] ‘After 40 years Thiele’s chronology has not been significantly altered or proved to be false in any major area except in the matter of Hezekiah’s coregency.’, McFall, ‘A Translation Guide to the Chronological Data in Kings and Chronicles’, Bibliotheca Sacra (148.589.42), 1996.

[16]It remained then for others to complete the application of principles that Thiele used elsewhere, thereby providing a chronology for the eighth-century kings of Judah that is in complete harmony with the reign lengths and synchronisms given in 2 Kings and 2 Chronicles. The most thorough work in this regard was Leslie McFall’s 1991 article in Bibliotheca Sacra.22 McFall made his way through the reign lengths and synchronisms of Kings and Chronicles, and using an exact notation that indicated whether the years were being measured according to Judah’s Tishri years or Israel’s Nisan years, he was able to produce a chronology for the divided monarchies that was consistent with all the scriptural texts chosen.’, ibid., pp. 105-106.

[17] ‘In a 1996 article, Kenneth Strand wrote, “What has generally not been given due notice is the effect that Thiele’s clarification of the Hebrew chronology of this period of history has had in furnishing a corrective for various dates in ancient Assyrian and Babylonian history.”28 The purpose of Strand’s article was to show that Thiele’s methodology accomplished more than just producing a coherent chronology from scriptural data. His chronology, once produced, proved useful in settling some troublesome problems in Assyrian and Babylonian history.’, Young, ‘Inductive And Deductive Methods As Applied To OT Chronology’, Master’s Seminary Journal (18.1.112-113), 2007.

[18] ‘Grabbe suggests that the names and sequence of kings in Israel and Judah, and their approximate chronological placement, agrees with what can be gleaned from extra-biblical sources. To this extent the biblical framework (meaning primarily 1 and 2 Kings) is reliable: even if we had no external sources we could have reasonable confidence in the biblical sequence of Jeroboam I, Nadab, Baasha, Elah, Omri, Ahab, Jehu, etc. in Samaria, and David, Solomon, Rehoboam, Abijam, Asa, Jehoshaphat, etc. in Jerusalem, along with their interrelationships. Beyond that it starts to get more and more tricky, with decreasing reliability in the biblical narrative as the detail increases (this is a general statement, and there are sometimes exceptions in specific instances).’, Grabbe, ‘Reflections on the Discussion’, Grabbe (ed.), ‘Ahab Agonistes: The Rise and Fall of the Omri Dynasty’, p. 337 (2007).

h1

Was the Genesis flood narrative copied from Mesopotamian myths?

June 1, 2011

The Challenge

By the end of the 19th century archaeology had discovered many Mesopotamian texts containing creation and flood narratives remarkably similar to those in the Bible.

Critical scholars came to believe that the Biblical narratives had simply been copied from earlier Mesopotamian myths.[1] [2]The Biblical flood narrative in particular is still considered by some scholars to have been borrowed from the Mesopotamian story.[3] [4]

The Facts

Later scholarship noted significant differences between the Biblical and Mesopotamian narratives;[5] the Mesopotamian creation narratives were now viewed as parallels to the Genesis narrative.[6]  Still later it was the Genesis and Babylonian accounts shared an earlier Mesopotamian source, whether literary or oral.[7] [8] [9]

Scholarly Views

Kitchen (Assyriologist), note that Assyrologists have abandoned the idea of Genesis 1-11 being borrowed from Assyrian and Babylonian texts.[10] Millard (Professor Emeritus of Hebrew and Ancient Semitic Languages), observes there is no evidence for direct literary borrowing.[11] [12] This is the majority view of current scholarship. [13]  [14] [15] [16] [17] [18] [19] [20]

At least early as 1872, it was suggested that the similarities between the Genesis and Mesopotamian flood narratives are due to the texts describing the same genuine historical event. [21]

In the early 20th century, critical scholar Hermann Gunkel observed that this was supported by the curious description (in both the Genesis account and the earlier Mesopotamian accounts), of the Ark being driven upstream, contrary to expectation.[22]  This explanation remains well represented in scholarship. [23] [24] [25] [26] [27]


[1] ‘Some argued that many Hebrew ideas actually originated in Mesopotamia and were borrowed by Israel.’, Chavalas, ‘Mesopotamia and the Bible’, p. 32 (2003).

[2] ‘The idea of Babylonian primacy was perfected by Delitzsch in 1902-1903. In his lectures, he argued that Israel could only be studied in light of Babylonia, and in fact Israelite civilization was derived from Babylonia.’, ibid., p. 32.

[3] ‘Since this portion of the biblical narrative postdates the Mesopotamian traditions (the final form of this portion of Genesis is usually dated to the fifth century B.C.E, although its oral or written sources may be dated as much as six hundred years earlier), it is conceivable, if not likely, that the biblical writer has borrowed and adapted Mesopotamian flood traditions.’, Fant & Reddish, ‘Lost Treasures of the Bible: Understanding the Bible through Archaeological Artifacts in World Museums’, p. 25 (2008).

[4] ‘It is commonly accepted that parts of Genesis 1–11 show literary dependence, either directly or indirectly, on Mesopotamian literary tradition.187 The best test case would be the flood story in Genesis 69.’, Smith, ‘God in Translation: Deities in Cross-Cultural Discourse in the Biblical World’, p. 182 (2010).

[5] ‘As scholars studied the significant differences and omissions between the accounts, they concluded that neither the Mesopotamian nor the biblical author borrowed from the other.’, Couch, ‘The Fundamentals for the Twenty-First Century: Examining the Crucial Issues of the Christian Faith’, p. 177 (2000).

[6] ‘Nevertheless, it adds much that is significant for the Near Eastern mythological horizon, and perhaps even provides a number of interesting parallels to the motifs of the biblical paradise story as told in the second and third chapters of Genesis.’, Kramer, ‘Sumerian Myths and Epic Tales’, in Pritchard (ed.), ‘Ancient Near Eastern texts relating to the Old Testament’, p. 37 (1950).

[7] ‘The similarities in broad outline and in certain points of detail between the Gilgamesh and the Genesis and the Gilgamesh versions are too striking to be accidental. Both probably derive from a common older Mesopotamian tradition, fragments of which are preserved in the Sumerian version.’, Davidson, ‘Genesis 1-11’, Cambridge Bible Commentaries p. 65 (1973).

[8] ‘It is undoubtedly borrowed from a common religious tradition of flood accounts.’, Brueggemnann, ‘Genesis’, p. 73 (1982).

[9] ‘Although the differences between the two stories may be too great to support a theory of direct literary dependence, most scholars are convinced that the biblical flood narrative is to some degree dependent upon ancient Mesopotamian flood narratives.’, Fant & Reddish, ‘Lost treasures of the Bible: understanding the Bible through archaeological’, p. 21 (2008).

[10]Thus most Assyriologists have long since rejected the idea of any direct link between Gen. 1-11 and Enuma Elish, and nothing else better can be found between Gen. 1-11 and any other Mesopotamian fragments.’, Kitchen, ‘On the Reliability of the Old Testament’, p. 424 (2003); his footnote reads ‘Assyriologists generally reject any genetic relationship between Gen. 1-2 and the Mesopotamian data because of the considerable differences; see (eg.) J.V. Kinnier-Wilson. In D. W. Thomas, ed., Documents from Old Testament Times (London: Nelson, 1958), 14; W. G. Lambert, JTS. n.s., 16 (1965): 287-300, esp. 289. 291, 293-99. and in ISF, 96-113, with addenda; A. R. Millard, TynB 18 (1967): 3-4.7. 16-18, and in ISIF 114-28; T. Jacobsen, in JBL 100 (198 1): 513-29, and translation, both now in ISIF 129-42, plus 160-66.’, ibid., p. 591.

[11]However, it has yet to be shown that there was borrowing, even indirectly. Differences between the Babylonian and the Hebrew traditions can be found in factual details of the Flood narrative (form of the Ark; duration of the Flood, the identity of the birds and their dispatch) and are most obvious in the ethical and religious concepts of the whole of each composition. All who suspect or suggest borrowing by the Hebrews are compelled to admit large-scale revision, alteration, and reinterpretation in a fashion that cannot be substantiated for any other composition from the ancient Near East or in any other Hebrew writing. If there was borrowing then it can have extended only as far as the “historical” framework, and not included intention or interpretation.’, Millard, ‘A New Babylonian “Genesis” Story’, in  Hess & Tsumura (eds.), ‘I Studied Inscriptions from Before the Flood: Ancient Near Eastern, Literary Approaches to Genesis 1-11’, Sources for Biblical and Theological Study, volume 4, p. 127 (1994).

[12] ‘The two accounts undoubtedly describe the same Flood, the two schemes relate the same sequence of events. If judgment is to be passed as to the priority of one tradition over the other, Genesis inevitably wins for its probability in terms of meteorology, geophysics, and timing alone.’, ibid., pp. 127-128.

[13] ‘The similarities between the Genesis account and the ‘Atra-Hasis Epic’ do not support the idea that Genesis is a direct borrowing from the Mesopotamian but do indicate that Mesopotamian materials could have served as models for Genesis 1-11, as Jacobsen holds. P.D. Miller also admits that ‘there were Mesopotamian models that anticipate the structure of Genesis 1-11 as a whole.’, Tsumura, ‘Genesis and Ancient Near Eastern Stories of Creation and Flood’, in ibid., p. 47.

[14] ‘With Genesis 1-11 we seem to be working more with shared motifs and basic plotlines that originated in Mesopotamia rather than with actually known texts directed [sic] borrowed into Israel.’, Smith, ‘God in Translation: Deities in Cross-Cultural Discourse in the Biblical World’, p. 182 (2010).

[15] ‘The Bible’s accounts of the creation of the world, the creation of humankind, and the flood were not borrowed from these, but neither are they unique in every respect.’, Arnold & Beyer (eds.), ‘Readings from the ancient Near East: primary sources for Old Testament study’, p. 13 (2002).

[16] ‘The details are not exact and most scholars deny any direct literary dependence but it would seem that both stories emerge from a common tradition or milieu.’, Moyise, ‘Introduction to Biblical Studies’, p. 33 (2004).

[17] ‘The Biblical flood of Noah in the book of Genesis 6-9 shares continuity with the other Ancient Near Eastern flood stories, but is probably not directly dependent on any of them.’, Snell, ‘A Companion to the Ancient Near East’, p. 256 (2005).

[18] ‘But after a careful study of the two, Alexander Heidel has concluded that “no incontrovertible evidence can for the present be produced” in favor of biblical dependence on the Babylonian materials. His conclusion regarding the flood accounts is similar.’, Niehaus, ‘Ancient Near Eastern Themes in Biblical Theology’, p. 22 (2008).

[19] Text

[20] ‘Many who have done thorough linguistic and literary analysis (e.g., A. Heidel, A.R. Millard, D. Damrosch) conclude that literary dependence cannot be demonstrated. Here, as in most of the parallels in the primeval history, it is considered more likely that Mesopotamian and biblical traditions are based on a common source. Some understand this common source to be a piece of more ancient literature, while others consider it the actual event.’, Hill & Walton, ‘A Survey of the Old Testament’, p. (2010).

[21] ‘Among many theorists, George Smith in 1872 [33] famously linked the great Biblical Flood of the book Genesis to an historical event, probably of the 3rd millennium BC, which deposited a 50-cm- sediment-layer in the Mesopotamian lowland.’, Haigh & Křeček, ‘Environmental Reconstruction in Headwater Areas’, p. 14 (2000).

[22] ‘The most characteristic element of the Babylonian account seems to be that the Ark, driven from the South inland against the current of the rivers, was stranded in the northern mountains. This element is so remarkable that it could only have been stimulated by a corresponding natural phenomenon. E. Suss (25ff.) suspects that a violent earthquake in the Persian Gulf may have been the cause. A powerful cyclone from the South, associated with voluminous rain and horrible darkness, drove the destructive waters far into the inhabited land. This event must have taken place in a very ancient time. The news of the terrible catastrophe was preserved through all times. This theory is certainly very plausible.’, Gunkel ‘Genesis’ (1910), Biddle (trans.), p. 77 (1997 English ed.).

[23] ‘This suggests that we are not dealing with a literary dependence or even a tradition dependence as much as we are dealing with two literary perspectives on a single actual event.‘, Walton, ‘Ancient Israelite Literature in Its Cultural Context: A Survey of Parallels’, p. 40 (1994).

[24] ‘The story may have arisen from a specific historical flood that took place in parts of southern Mesopotamia around 2900.’, Tigay, ‘The Evolution of the Gilgamesh Epic’, p. 214 (2002).

[25] ‘Could not stories be shared by the Bible and surrounding cultures because they are both based on a historical event? Both Scripture and Mesopotamian literature mention a flood because there indeed was a flood.’, Hamilton, ‘Handbook on the Pentateuch: Genesis, Exodus, Leviticus, Numbers, Deuteronomy’, p. 66(2005).

[26] ‘However, there are more options than simply concluding that the Bible borrowed from Babylon. An equally plausible explanation is that both traditions go back to a real event.’, Longman, ‘How to read Genesis’, pp. 86-87 (2005).

[27] ‘On the basis of substantial historical evidence, coupled with many parallel words and phrases, what reasonable conclusions could we make? Here are just three: 1. There is a likelihood that a flood event actually happened. Why would the Akkadians, Sumerians, and Hebrews invent such a story unless there was some historical basis? 2. considering the parallel accounts are describing a historical event in the region of southern Mesopotamia about 2900 B.C., then Genesis also is describing the same historical, regional flood, and not a global deluge. 3. A regional flood would have brought judgment to those in the region. Judgment would have been specific to the sinful Adamite population, those answerable to God, rather than a universal pronouncement upon all mankind everywhere.’, Fischer, ‘Historical Genesis: from Adam to Abraham’, p. 140 (2008).

h1

Is Solomon’s wealth a literary fiction?

April 30, 2011

The Challenge

The Biblical account of Solomon’s wealth has been described as unrealistic, in standard critical commentaries.[1] Many scholars are sceptical, [2] [3] [4] though some express their doubts cautiously.[5] [6]

Ancient Uses of Gold

Ancient uses of gold for construction which are analogous to Solomon’s include the tomb of Tutankhamen,[7] extensive use of gold plating on buildings in the reign of Tuthmosis III,[8] massive gold use on buildings of the Egyptian New Kingdom era,[9] and the same kind of gold usage in Babylonia and Assyria.[10] Millard also points out that items described as ‘of gold’ were not always solid gold; often they were covered in gold plate or gold leaf.[11]

Solomon’s Income

The Bible identifies ‘Ophir’ as one source of Solomon’s gold.[12] Although the location of Ophir is unknown, archaeological evidence identifies it as a source of gold.[13] Solomon’s income of 666 talents of gold in one year[14] [15]  is considered fictional by some commentators.[16]

Although this income is unique in Ancient Near East records,[17] the 120 talents of gold received by Solomon from Tyre[18] is matched and exceeded by gifts and tribute of gold from other Ancient Near East monarchs.[19] [20] [21] [22] [23]

The vast gold expenditure of pharaoh Orsokon I exceeded even Solomon’s, [24] and it is likely his wealth was the result of his father Sheshonq’s conquest of Solmon’s son Rehoboam.[25] [26] [27] [28]


[1]The gilding of the furnishings, as of the altar, is reasonable, but not that of the whole interior; cf. Stade, and Nowack, Arch., 2, 29, n. I.’, Montgomery, ‘A Critical and Exegetical Commentary on the Book of Kings’, p. 152 (1951).

[2]Such extravagant description appears to be a step forward in the process of exuberant imagination, continued by the Chronicler, for whose fancy even the 120-cubit high portico was overlaid with fine gold (2 Ch. 34ff.).’, ibid., p. 152.

[3] ‘Some have regarded this description as exaggerated.’, Hicks, ‘1 & 2 Chronicles’, College Press NIV Commentary, p. 306 (2001).

[4] ‘Some have questioned the authenticity of this description, labeling it unabashed exaggeration.’, Long, ‘1 & 2 Kings’, College Press NIV Commentary, p. 147 (2002).

[5]Despite all exaggerated accounts of Solomon’s wealth and commercial success, which were written to give him honor and prestige, there is an historical kernel in the reports of his wealth.’, Esler, ‘Ancient Israel: The Old Testament in its social context’, p. 105 (2006).

[6] ‘Evidently, we can not take the figures about Solomon’s mercantile activities and revenues given in the account at face value. They must have been fabulously exaggerated. Nevertheless, in Ishida’s assessment, which I share, “We can hardly deny the substantial historicity comprised in them” (p. 109).’, Corral, ‘Ezekiel’s Oracles Against Tyre: historical reality and motivations’, p. 112 (2002).

[7] ‘Here were many articles of furniture plated with sheets of gold, beaten and engraved, a wealth of elaborate golden jewellery, a golden dagger, the king’s gold mask, and, eclipsing all, his coffin of solid gold.7 Its weight is 110.4 kg (243 lbs). Particularly relevant for the present study are the shrines that stood in the tomb. There is a small wooden shrine (50 cms high, 26.6 cms wide, 32 cms deep, 19¾ x 10½ x 12¾ inches) made to hold a statue. Sheets of gold cover it entirely, within and without, embossed and engraved with scenes of the king’s life, magical figures, and inscriptions.’, Millard, ‘Solomon In All His Glory’, Bible and Spade (11.2-3-4.64-65), 1982.

[8] ‘In the Temple of the Sacred Boat at Karnak stood twelve columns erected by Tuthmosis III, about 1450 BC, each about 3½ metres high, designed to represent bundles of papyrus. Each was entirely covered with gold, fastened in slits cut at suitable points in the pattern. In another hall at Karnak were fourteen columns. Their design was similar, a papyrus stem, and they, too, were plated with gold from top to bottom. These pillars were larger; an inscription states that they were 31 cubits, that is 16.25 metres high (53 feet).’, ibid., p. 67.

[9] ‘Tuthmosis III (c. 1490–1436 BC) recorded his building of a shrine ‘plated with gold and silver’, and of a floor similarly made. Amenophis III in the next century decorated several structures in this way. Of one temple in honour of Amun at Thebes he claimed it was ‘plated with gold throughout, its floor is adorned with silver, all its portals with electrum’, while the temple at Soleb had the same treatment, except that ‘all its portals are of gold’. Ramesses II (c. 1297–1213 BC) provided his mortuary temple at Abydos with doors ‘mounted with copper and gilded with electrum’. Later in this period, Ramesses III (c. 1183–1152 BC) ornamented temples in exactly the same way. At Medinet Habu he constructed a shrine of gold with a pavement of silver, and doorposts of fine gold.’, ibid., p. 68.

[10] ‘Esarhaddon of Assyria (680–669 BC) told how he restored the shrine of his national god, Assur, and ‘coated the walls with gold as if with plaster’. His son Ashurbanipal claimed much the same, ‘I clad its walls with gold and silver’. In Babylon a century later Nebuchadnezzar recorded his enrichment of the shrines of his gods, ‘I clad (them) in gold, and made them bright as day’, and Nabonidus (555–539 BC) followed him, ‘I clad its walls with gold and silver, and made them shine like the sun’. The tradition stemmed from much earlier times in Babylonia, for Entemena of Lagash built a temple for his god ‘and covered it with gold and silver’ about 2400 BC.16’, ibid., pp. 68-69.

[11]While words like ‘a gold statue’ or ‘a gold bed’ in ancient documents should not be pressed to mean ‘made of solid gold throughout’ or ‘the purest gold’, they can be understood to mean ‘gold all over’, that is to say, nothing else could be seen.’, ibid., pp. 69-70.

[12] 1 Kings 9: 28 They sailed to Ophir, took from there four hundred twenty talents of gold, and then brought them to King Solomon.

[13]The expression “gold of Ophir” occurs not only in the Bible, but also on an eighth-century B.C. ostracon* found at Tell Qasile in Israel. That ostracon, while showing that the name was current to designate the origin or type of gold, throws no light on Ophir’s location.’, Millard, ‘Does the Bible Exaggerate King Solomon’s Golden Wealth?’, Biblical Archaeology Review (15.03), May/June 1989.

[14] 1 Kings 10:14 Solomon received 666 talents of gold per year; commentators are divided as to whether this represents an annual income, or the income of one particular year.

[15] ‘On the basis of these figures, Solomon’s gold can be computed as: 120 talents==3,960 kg==3.9 tons from Tyre, and the same from Sheba; 420 talents==13,860 kg==13.6 tons from Ophir; 666 talents==21,978 kg==21.6 tons in one year.’, Millard, ‘Solomon In All His Glory’, Bible and Spade (11.2-3-4.72), 1982.

[16] ‘Indeed, J.B. Pritchard argues that the narrative’s references to gold, pure gold and silver and its allusions to the respect which Solomon’s peers showed to him are ‘popular—even folkloristic’ elements of the history of Solomon’s age.’, Younger Jr, ‘The Figurative Aspect and the Contextual Method in the Evaluation of the Solomonic Empire: 1 Kings 1–11’, in Clines, Fowl & Porter (eds.), ‘The Bible in Three Dimensions: Essays in Celebration of Forty Years of Biblical Studies in the University of Sheffield’, p. 159 (1990).

[17]The only ancient text that reports the annual income of a powerful king in Old Testament times is the Hebrew Bible. In 1 Kings 10:14 the figure of 666 talents of gold (almost 25 U.S. tons) is given for Solomon. This may refer to a particular year, just as the 420 talents (15.75 U.S. tons) from Ophir refers to a particular source (1 Kings 10:11). Only two figures in ancient records approach the amount of 666 talents: the total of Pharaoh Osorkon’s gift to the gods and the amounts of treasure Alexander the Great found in Persia.’, Millard, ‘Does the Bible Exaggerate King Solomon’s Golden Wealth?’, Biblical Archaeology Review (15.03), May/June 1989.

[18] 1 Kings 9: 28 They sailed to Ophir, took from there four hundred twenty talents of gold, and then brought them to King Solomon.

[19] ‘We learn from firsthand sources that Metten II of Tyre (ca. 730) paid a tribute of 150 talents of gold to our old acquaintance Tiglath-pileser III of Assyria, while in turn his successor Sargon II (727-705) bestowed 154 talents of gold upon the Babylonian gods – about 6 tons in each case. Going back almost eight centuries, Tuthmosis III of Egypt presented about 13.5 tons (well over 200 talents) of gold in nuggets and rings to the god Amun in Thebes, plus an unknown amount more in a splendid array of gold vessels and cult implements. Worth almost a third of Solomon’s reputed annual gold revenue, this was on  just one occasion, to just one temple.’, Kitchen, ‘On the Reliability of the Old Testament’, pp. 133-134 (2003).

[20] ‘So a king of Assyria wrote to the Pharaoh about 1350 BC, ‘Gold is like dust in your land, one simply gathers it up.’ A contemporary king repeated this statement six times in letters to the Pharaoh! The Assyrian went on ‘Why do you think it is so valuable? I am building a new palace, send me enough gold to decorate it properly! When my ancestor wrote to Egypt, he was sent twenty talents of gold. . . . When (another king) wrote to Egypt to your father, he sent him twenty talents of gold . . . send me much gold!’ (Twenty talents by Babylonian standards was 600 kg or 11.7 cwts.).’, Millard, ‘Solomon In All His Glory’, Bible and Spade (11.2-3-4.73), 1982.

[21] ‘When Damascus surrendered to Adadnirari III, probably in 796 BC, the Assyrian received 2,300 talents of silver (69,000 kg; 67.76 tons), 20 talents of gold (600 kg; 1,320 lbs), and much else. Some sixty years later Tiglath-pileser III subjugated Samaria, placing Hoshea on the throne as his nominee. Samaria paid 10 talents of gold (300 kg; 660 lbs) as tribute (and an unknown amount of silver). The same emperor received the submission of Tyre, and with it the large sum of 150 talents of gold (4,500 kg; 4.4 tons).’, ibid., p. 74.

[22] ‘During the reign of Tuthmosis III the yield of the gold fields at Wawat in Nubia (the Sudan) for three years was 232.4 kg (512 lbs), 258.8 kg (570 lbs), and 286.1 kg (630 lbs). These may be exceptional figures, yet they show what sort of income was available from a single source. In the Annals of the same pharaoh, the booty taken between his twenty-second and his forty-second years amounted to over 11,500 kg (11.3 tons) of gold. His successor, Amenophis II (c. 1427–1401 BC) claimed the weight of gold vessels he took from the Levant was 6,800 deben (618.5 kg; 1,360 lbs).’, ibid., p. 75.

[23] ‘None of these figures approach the amounts recorded for Solomon except for the booty gathered by Tuthmosis III (11,500 kg; 11.3 tons).’, ibid., p. 75.

[24] ‘In Egypt Shishak’s successor Osorkon I gifted some 383 tons of gold and silver to the gods and temples of Egypt in the first four years of his reign, many of the detailed amounts being listed in a long inscription (now damaged) (figs. 22A, B). That sum would (in weight) be equivalent to almost seventeen years of Solomon’s annual gold revenue,’, Kitchen, ‘On the Reliability of the Old Testament’, p. 134 (2003).

[25] 1 Kings 1425 In King Rehoboam’s fifth year, King Shishak [Sheshonq] of Egypt attacked Jerusalem. 26 He took away the treasures of the LORD’s temple and of the royal palace; he took everything, including all the golden shields that Solomon had made.

[26]His reign is poorly documented, nothing hints at a far-reaching military adventure, bringing home rich booty.’, Millard, ‘Solomon In All His Glory’, Bible and Spade (11.2-3-4.76), 1982.

[27] ‘Osorkon’s father was Sheshonq 1 (c. 945–924 BC), the Shishak who took the gold from Solomon’s Temple in Jerusalem and from the Judaean treasury.’, ibid., p. 76.

[28] ‘Where could Osorkon have obtained such immense wealth, to spend on such a scale after only three and a third years of his reign? Barely five years earlier, Osorkon’s father Shishak had looted the wealth of Jerusalem. It seems unlikely to be a mere coincidence that almost immediately after that event Osorkon could dispose so freely of so much gold and silver.*The vast amounts of Solomon’s golden wealth may have ended up, at least in part, as Osorkon’s gift to the gods and goddesses of Egypt.’, Kitchen, ‘Where Did Solomon’s Gold Go?’, Biblical Archaeology Review (15.03), May/June 1989.

h1

What is archaeological ‘minimalism’?

April 26, 2011

Definition

The ‘minimalist’ view is that archaeology provides little or no support for the Biblical history.[1] [2] The best known adherents are Philip Davies,[3]  Lester Grabbe,[4] Niels Lemche,[5] Thomas Thompson,[6] and Keith Whitelam.[7] However, Israel Finkelstein and Neil Silberman are the only two prominent archaeologists associated with minimalist views.

Kenneth Kitchen

Kitchen[8] has raised numerous objections to minimalist claims, rejecting Thompson’s assertion that the Hebrew Tabernacle is a literary fiction,[9] that the Merneptah Stele is not reliable evidence for a people named ‘Israel’ in early 13th century Canaan,[10] that the Tel Dan Stele does not refer to a Hebrew ‘House of David’,[11] that the description of Solomon’s wealth is legendary,[12] and that the use of the first person perspective in the Mesha Stele indicates a post-mortem or legendary account.[13] [14] [15] Kitchen has also criticized Finkelstein and Silberman.[16] [17]

William Dever

Though far more sceptical than Kitchen, Dever[18] has nevertheless opposed minimalism vigorously. [19] [20] [21] [22]

Israel Finkelstein

Despite sympathies with some minimalist views, Finkelstein has rejected strongly the minimalist claims concerning Persian era Hebrew scribes,[23] that the ‘lists and details of royal administrative organization in the kingdom of Judah’ are fictional,[24] and that the Hebrew King David never existed.[25] [26] He has also acknowledged strong archaeological support for certain parts of the Biblical record.[27]

Amihai Mazar

With more in common with Finkelstein than the minimalists,[28] Mazar[29]  takes a moderate though critical view of the Biblical history.[30] [31]


[1] The ‘maximalist’ view is that archaeology overwhelmingly supports the Biblical history, and the moderate view is that archaeology substantially supports the Biblical history but that not all of the history can be supported directly from archaeology.

[2] ‘A recent trend in Syro-Palestinian archaeological study known as minimalism or revisionism suggests that Israel was created in the Hellenistic period.’, McCarty & Merrill, ‘Review: What Did the Biblical Writers Know and When Did They Know It? By William G. Dever.’, Bibliotheca Sacra (161.641.115), 2004.

[3] Biblical scholar.

[4] Historian.

[5] Biblical scholar.

[6] Biblical scholar.

[7] Biblical scholar.

[8] Egyptologist, Assyriologist, and archaeologist.

[9] ‘In so doing he ignores the whole of the comparative data that show clearly that the tabernacle was a product of Egyptian technology from the overall period 3000 to 1000 D.C. (plus Se-mitic analogues, 1900-1100), and would be unable to account for such facts.’, Kitchen, ‘On the Reliability of the Old Testament’, pp. 450-451 (2005).

[10] ‘The Israel of Merenptah’s stela was, by its perfectly dear determinative, a people (= tribal) grouping, not a territory or city-state; rare statements to the contrary are perverse nonsense, especially given the very high level of scribal accuracy shown by this particular monument.’, ibid., p. 451.

[11] ‘(i) The name “David” may be unusual, but is not unparalleled. Long centuries before, it was borne by a West Semitic chief carpenter in about 1730 B.C. on an Egyptian stela formerly in the collection at Rio de Janeiro. (ii) Dwd is neither the name (which Thompson admits) nor an epithet of a deity. Others are beloved of deities (for which references are legion!), but male deities are not beloved of others, human or divine (only goddesses are beloved of their divine husbands in Egypt). (iii) Mesha’s stela is ninth, not eighth, century. (iv) On Mesha’s stela dwd(h) is not a divine epithet of YHWH or anyone else. (v) Contrary to TLT, “House of X” does  mean a dynastic founder, all over the Near East, in the first half of the first millennium B.C.; it was an Aramean usage that passed into Assyrian nomenclature, and examples are common. (vi) Again, the expression, in part of its usage, is like the British “House of Windsor”, etc. Such usages were not peculiar to Aram, Assyria, and Judah either: in Egypt, the official title given to the Twelfth Dynasty (Turin Canon) was “Kings of the House (lit. ‘Residence’) of Ithet-Tawy” = ‘the Dynasty of Ithet-Tawy”. And the Thirteenth Dynasty was duly entitled “Kings who came after the [House of] King Sehetepibre” (founder of the Twelfth Dynasty). (vii) The charge of forgery is a baseless slur against the Dan expedition, without a particle of foundation in fact.’, ibid., pp. 452-453.

[12] ‘The point of the comparisons drawn with external (and firsthand!) sources was precisely that Solomon’s wealth (even as stated in Kings) was not exceptional or “fabulous/legendary” in its wider context. He was a pauper compared with (e.g.) Osorkon I, who, less than a decade after Solomon’s death, spent sums that massively outstrip Solomon’s stated income, and gave detailed accounts. The layering that TLT objects to was customary. At Karnak in Egypt, some temple columns were grooved to fit sheet gold from top to bottom, not mere “plastering.” As a touch of throwaway wealth, one need look no further than the recently discovered burials of two Assyrian queens. Solomon had just one golden throne? One pharaoh was sent ten at a time!’, ibid., p. 454.

[13] ‘Use of the first person by a monarch does not belong exclusively to either postmortem memorial texts or to later legends about such kings. A huge army of texts shows up the falsity of his presumption.’, ibid., p. 456.

[14]For first person, not postmortem, see (e.g.) Lipit-Ishtar (247), Warad-Sin/Kudur-mabuk (251-52), Rim-Sin I (253, his third text), Hammurabi (256-57), Ammi-ditana (258-59), and Shamshi-Adad I (259), all early second millennium. In the first millennium every major Assyrian king did exactly likewise, in various editions of their annals that were anything but postmortem, from Tiglath-pileser I to Assurbanipal (cf. ANET, 274-301; CoS II,261-306; RIMA, 1-3 passim).’, ibid., p. 456.

[15]Mesha’s stela is a contemporary building plus victory text, exactly like so many other inscriptions in the ancient Near East in all places and periods; the first-person formulation is irrelevant to its status, as the foregoing examples (far from exhaustive!) show.’, ibid., p. 457.

[16] ‘His reevaluation of the realm of Omri and Ahab is refreshing but wildly exaggerated, especially in archaeological terms. As others have shown amply, the redating will not work (cf. chap. 4, sec. 3 above).’, ibid., p. 464.

[17]The Philistines of Gerar (not those of the Pentapolis!) are a very different lot from the Iron Age group of that name. The term is a probably twelfth-century one substituted for Caphtorim or the like, precisely as Dan was substituted for Laish in Gen. 14:]4.’, ibid., p. 465.

[18] Archaeologist.

[19] ‘There are some who claim that the Bible contains little or no historical information about ancient Israel. I want to combat these “minimalist” or “revisionist” views of the history of ancient Israel by showing how archaeology can and does illuminate a historical Israel in the Iron Age of ancient Palestine (roughly 1200–600 B.C.E.)’, Dever, ‘Save Us from Postmodern Malarkey’, Biblical Archaeology Review (26.02), March/April 2000.

[20]Davies does not even cite the standard handbook, Mazar’s Archaeology of the Land of the Bible,’, Dever, ‘Who Were the Early Israelites? And Where Did they Come From?’, p. 138 (2003).

[21] ‘That same year Whitelam wrote an article for the Sheffield Journal for the Study of the Old Testament on “the realignment and transformation of Late-Bronze-Iron Age Palestine.” It was so full of caricatures of modern archaeological theory and results that I felt compelled to answer it in the same journal.’, ibid., p 139.

[22] ‘Thus he [Thompson] published two years later his revisionist treatment of ancient Israel: The Mythic Past: Biblical Archaeology and the Myth of Israel. Despite its subtitle, this work has next to nothing to do with real archaeology.’, ibid., p. 141.

[23] ‘First of all, as the biblical scholar William Schniedewind has indicated, literacy and extensive scribal activity in Jerusalem in the Persian and early-Hellenistic periods were much less influential than in the seventh century b.c.e. The assumption is inconceivable that in the fifth, or fourth, or even second centuries b.c.e., the scribes of a small, out-of-the-way temple town in the Judean mountains authored an extraordinarily long and detailed composition about the history, personalities, and events of an imaginary Iron Age “Israel” without using ancient sources.’, Finkelstein, ‘Digging for the Truth: Archaeology and the Bible’, in Schmidt (ed.), ‘The Quest for the Historical Israel’, Archaeology and Biblical Studies, number 17, p. 13 (2007).

[24] ‘The sheer number of name lists and details of royal administrative organization in the kingdom of Judah that are included in the Deuteronomistic History seems unnecessary for a purely mythic history. In any event, if they are all contrived or artificial, their coincidence with earlier realities is amazing.’, ibid., p. 13.

[25] ‘This argument suffered a major blow when the Tel Dan basalt stele was discovered in the mid-1990s.’, ibid., p. 14.

[26] ‘This was the first time that the name “David” was found in any contemporary source outside the Bible, in this case only about a century after his own supposed lifetime.’, ibid., p. 14.

[27] ‘Archaeological excavations and surveys have confirmed that many of the Bible’s geographical listings—for example, of the boundaries of the tribes and the districts of the kingdom—closely match settlement patterns and historical realities in the eighth and seventh centuries b.c.e. Equally important, the biblical scholar Baruch Halpern showed that a relatively large number of extra-biblical historical records—mainly Assyrian—verify ninth- to seventh-century b.c.e. events described in the Bible: the mention of Omri in the Mesha stele, those of Ahab and Jehu in the Shalmaneser III inscriptions, Hezekiah in the inscriptions of Sennacherib, Manasseh in the records of Esarhaddon and Ashurbanipal, and so on. No less significant is the fact, as indicated by the linguist, Avi Hurwitz, that much of the Deuteronomistic History is written in late-monarchic Hebrew, which is different from the Hebrew of post-exilic times.’, ibid., pp. 13-14.

[28]‘ Our views differ on certain important issues, but we share more in common than we do with either of the two extreme groups described above.’, Mazar, ‘On Archaeology, Biblical History, and Biblical Archaeology’, in Schmidt (ed.), ‘The Quest for the Historical Israel’, Archaeology and Biblical Studies, number 17, p. 29 (2007).

[29] Archaeologist.

[30] ‘My own choice is to follow those who claim that the initial writing of the Torah (the Pentateuch or Tetrateuch), of the Deuteronomistic History and large parts of the prophetic and wisdom literature took place during the late monarchy (eighth to early-sixth centuries b.c.e.), while during the exilic and post-exilic periods they underwent further stages of editing, expansion, and change. Yet, I also accept the

view of many scholars that the late-monarchic authors utilized earlier materials and sources.’, ibid., p. 29.

[31] ‘Both Assyrian inscriptions and local inscriptions like the stelae of Mesha, king of Moab, and of Hazael, king of Damascus (better known as the Tel Dan inscription), confirm that the general historical framework of the Deuteronomistic narrative relating to the ninth century was based on reliable knowledge of the historical outline of that century. Our understanding of the periods preceding the ninth century is of course foggier.’, ibid., p. 30.

h1

Were camels domesticated in the time of Abraham?

April 24, 2011

The Challenge

WF Albright, one of the most famous 20th century archaeologists, argued that the camel was not domesticated until around the 1st millennium, well after the time of Abraham.[1] This was considered persuasive by many Biblical scholars, who were convinced that references in Genesis to camels in Egypt during the time of Abraham[2] are anachronistic. [3] [4] [5]

The Evidence

Some evidence alleged for very early camel use in Mesopotamia has proved dubious,[6] [7] but Albright overlooked evidence for camel domestication reported by the French archaeologist Petrie in 1907.[8]

However, Petrie’s evidence for camel domestication during the Ramesside era in Egypt (1292-1069 BCE), was still too late for Abraham (from around 1900 BCE), though significantly earlier than Albright’s date.

Evidence for early camel domestication elsewhere in the Ancient Near East and North Africa is well documented, [9] [10] [11] [12] and has been used to argue defend the Genesis account. [13] [14] [15] [16]

It is recognized domesticated camel caravans must have passed through Egypt at an early date, even though the Egyptians made no reference to them at this time. [17] [18]  Bulliet observes that evidence for the early domestication of the camel in Mesopotamia cannot be ignored on the basis of their absence in Egyptian evidence.[19]

He agrees with Albright that evidence for Syrian domestic camel use during the 3rd to 2nd millennium is absent,[20] and argues the undisputed evidence of their use elsewhere in Mesoptamia indicates they entered the area on a very small scale as pack animals by rich traders, rather than being herded in large numbers.[21] [22]

Firm evidence for very early camel domestication in Egypt has caused some scholars to reconsider the Biblical narrative.[23] [24] [25]


[1]According to Albright, any mention of camels in the period of Abraham is a blatant anachronism, the product of later priestly tampering with the earlier texts in order to bring more in line with altered social conditions. The Semites of the time of Abraham, he maintains, herded sheep, goats, and donkeys but not camels, for the latter had not yet been domesticated and did not really enter the orbit of Biblical history until about 1100-1000 BC with the coming of the Midianites, the camel riding foes of Gideon.’, Bulliet, ‘The Camel and the Wheel’, p. 36 (1990 ed., originally published 1975).

[2] Genesis 12: 15 When Pharaoh’s officials saw her, they praised her to Pharaoh. So Abram’s wife was taken into the household of Pharaoh, 16 and he did treat Abram well on account of her. Abram received sheep and cattle, male donkeys, male servants, female servants, female donkeys, and camels.

[3] ‘Some scholars have suggested that only with the 1st millennium B.C. was the camel fully domesticated’, Pratico, ‘Nomadism’, in Bromiley (ed.), International Standard Bible Encyclopedia, volume 3, p. 547 (rev. ed. 1988).

[4]The almost unanimous opinion of Biblical scholars is that mention of domesticated camels in the Patriarchal narratives (Gn 12:16; 24:10; 30:43) constitutes an anachronism. Camels, they say, were not domesticated until late in the second millennium BC, centuries after the Patriarchs were supposed to have lived.’, Caesar, ‘Bible and Spade (13.77), 2000.

[5] ‘There continue to be some scholars who follow Albright’s skepticism (1942; 1945; 1949: 207) that references to camels in the patriarchal narratives are anachronistic (e.g. Koehler-Rollefson 1993: 183).’,  Younker, ‘Bronze Age Camel Petroglyphs In The Wadi Nasib, Sinai’, Bible and Spade (13.75), 2000.

[6] ‘To be sure, one or two representations of camels from early Mesopotamia have been alleged, but they are all either doubtfully camelline, as the horsy looking clay plaque from the third dynasty of ur (2345-2308 B.C.), or else not obviously domestic and hence possibly depictions of wild animals,’, Bulliet, ‘The Camel and the Wheel’, p. 46 (1990 ed., originally published 1975).

[7] ‘These five pieces of evidence, needless to say, may not convince everyone that the domestic camel was known in Egypt and the Middle East on an occasional basis between 2500 and 1400 B.C. Other early depictions, alleged to be of camels, which look to my eyes like dogs, donkeys, horses, dragons or even pelicans, might be more convincing to some than the examples described above.’, ibid., p. 64.

[8]The pottery figure of a camel laden with water-jars was found in a tomb of the XIXth dynasty in the northern cemetery. There were no traces of a later re-use of the tomb; the style of the figure is of the rough fingered pottery of the XIXth dynasty, and quite unlike any of the moulded Roman figures; and the water-jar, is of the XIVIIIth-XIXth dynasty type and not of a form used in Greek or Roman times. Hence it is impossible to assign this to the age when the camel is familiar in Egypt, and it shows that as early as Ramesside times it was sufficiently common to be used as a best of burden.’, Petrie, ‘Gizeh and Rifeh’, in ‘Publications of the Egyptian Research Account and British School of Archaeology in Egypt’ (13.23), 1907.

[9] ‘Camels are not anachronistic in the early second millennium BC, but find only sparing attestation and use both in Genesis and external sources then and until the twelfth century BC.’, Kitchen, ‘Historical Method and Hebrew Tradition’, Tyndale Bulletin (17.1.83), 1966.

[10] ‘Both the dromedary (the one-humped camel of Arabia) and the Bactrian camel (the two-humped camel of Central Asia) had been domesticated since before 2000 BC.’, Scarre, ‘Smithsonian Timelines of the Ancient World’, p. 176 (1993).

[11] ‘As far as hard dates go, the 2500-1500 B.C. suggested earlier for the introduction of the camel into Somalia is the best that can be done from available data. Given the stage domestication had reached by the time the camels and their owners crossed the sea, some additional time must be allowed for earlier stages. Taking this into consideration, it is easily conceivable that the domestication process first got underway between 3000 and 2500 B.C.’, Bulliet, ‘The Camel and the Wheel’, p. 56 (1990 ed., originally published 1975).

[12]Found in a context datable to 2700 B.C., the remains led the excavators to argue that camel domestication began in Turkmenia and spread south (Compagnoni and Tosi 1978: 95–99). The domestic camel was apparently known to the inhabitants of the Indus Valley Civilization by 2300 B.C., although the species utilized remains open to question (Meadow 1984: 134 and references).’,  Zarins, ‘Camel’, in Freedman (ed.), Anchor Yale Bible Dictionary (electronic ed. 1996).

[13]Archeological discoveries have now shown clearly that references to domesticated camels in Genesis are by no means anachronistic, as some earlier scholars supposed. While camel caravans seem to have been used regularly only from the Late Bronze Age onward, archeologists have found numerous bones of domesticated camels. Thus when Parrot was excavating Mari, he found camel bones in the ruins of a house dated in the pre-Sargonic period (ca 2400 B.C.). An eighteenth-century-B.C. relief from Byblos pictured a camel in a kneeling position, and a socket on the back showed that the animal’s hump and its load had been attached separately. In accord with patriarchal traditions, cylinder seals from Middle Bronze Age Mesopotamia showed riders seated upon camels.’, Harrison, ‘Genesis’, in Bromiley (ed.), International Standard Bible Encyclopedia, volume 3, p. 547 (rev. ed. 1988).

[14] ‘Excavations in eastern Arabia, an area once believed to be a cultural backwater unworthy of archaeological investigation, have turned up evidence that camels were first domesticated by Semites before the time of Abraham. Much of this evidence has been examined by M. C. A. MacDonald of the Oriental Faculty at the University of Oxford’, Caesar, ‘Bible and Spade (13.77), 2000.

[15] ‘the principle area of extensive early camel domestication was the Syro-Arabian desert, due west of Ur, Abraham’s birthplace (1995: 1356).’, ibid., p. 77.

[16] ‘possession of camels by Semitic travelers endowed them with a special advantage over those who did not, particularly in economic and political terms. This conforms to the Genesis image of the Patriarchs as wealthy, respected individuals who could hold their own against monarchs and chieftains.’, ibid., p. 78.

[17]Horses and camels were not represented in Old Kingdom Egypt and camels are said to have been introduced into Egypt much later than horses.’, Daly, ‘Egyptology: the missing millennium: ancient Egypt in medieval Arabic writings’, p. 102 (2005).

[18] ‘In view of the very early caravan links between Arabia and the Nile Valley, it would be very surprising if the camel had not reached Egypt before the first millennium BC; doubtless there were religious reasons for the lack of representations of this animal earlier than this. Camels could have been first introduced to Egypt from 1680 BC by the invading Hyksos, but it is not until the end of the second millennium that references to them begin to be found;’, Fage, ‘The Cambridge history of Africa: From the earliest times to c. 500 BC’, volume 1, pp. 288-289 (1982).

[19]Yet it is very difficult to explain away all of the evidence pointing to the camel’s presence outside the Arabian peninsula prior to the year 1400B.C. The effort is better spent looking into the reasons why the evidence from this early period is so very scarce.’, Bulliet, ‘The Camel and the Wheel’, p. 36 (1990 ed., originally published 1975).

[20] ‘The archaeological record, as Albright affirms, shows no indication of camel use in the Syrian area during the period in question, 2500-1400B.C.,’, ibid., p. 64.

[21] ‘Indeed, they must have played little or no part in the ordinary herding economy of the time. The most satisfactory explanation of this circumstance is that the camel was known because it was brought into the area by traders carrying goods from southern Arabia but that it was not bred or herded in the area. It is worthy of note that whereas the citations from the Bible associating camels with Abraham and his immediate descendants seem to fit the generalized pattern of later camel use in the area, they could also fit a pattern in which camels were very uncommon. The largest number of animals mentioned in those episodes is ten, and those ten are probably most of what Abraham had’, ibid., pp. 64-54.

[22]But it has been demonstrated that the camel was already in use during the period in question and that its probable homeland was southern Arabia. It is much more reasonable, therefore, to assume that the camel was the main carrier on the incense route from the very beginning, or nearly so, and that the Semitic tribes of the north came to know the camel in this way in very small numbers. In other words, the presence of camels in the Abraham story can be defended and the story treated as primary evidence of camel use without disputing Albright’s contention that camel-breeding nomads did not exist in Syria and northern Arabia at that time.’, ibid., pp. 66-67.

[23] ‘However, in various parts of the country some evidence for the presence of camels has been uncovered, associated with dates as far back as the predynastic period (Free 1944:191).’, Daly, ‘Egyptology: the missing millennium : ancient Egypt in medieval Arabic writings’, p. 102 (2005).

[24] ‘In the Egyptian Fayum province was found a camel-skull dated to the ‘Pottery A’ stage, i.e. within the period c. 2000–1400 BC, the period from the Patriarchs practically to Moses; see O. H. Little, Bulletin de l’Institut d’Égypte 18, 1935–6, p. 215.’, Kitchen, ‘Camel’, in Wood & Marshall (eds.), ‘New Bible Dictionary’, p. 160 (3rd ed. 1996).

[25] ‘However, there is now a growing body of scholars who believe that camel domestication must have occurred earlier than previously thought (prior to the 12th century BC) and that the patriarchal narratives accurately reflect this (e.g., Ripinsky 1984; Coote and Whitelam 1987: 102; Zarins 1992: 826; Borowski 1998: 112–18).’, Younker, ‘Bronze Age Camel Petroglyphs In The Wadi Nasib, Sinai’, Bible and Spade (13.75), 2000.

h1

Did Luke use Josephus when writing Acts?

April 23, 2011

The Claim

It has been claimed that Luke used the writings of Josephus (specifically ‘Antiquities of the Jews’).[1] [2] Since Josephus wrote in 93 CE, this would date Acts no earlier than this time.[3] The following passages are claimed as examples of Luke’s dependence on Josephus.

*  Luke 3:1: Josephus and Luke record the census of Quirinius, but Luke’s differs from that of Josephus and cannot be verified independently; both Luke and Josephus refer to Lysanias the tetrarch of Abilene

*  Luke 13:1: Luke’s description of the murder of the Galileans is similar to Josephus’ description of an assault on Samaritans[4]

*  Acts 5:36-37: Luke mentions Theudas and Judas the Galilean, but reverses the order in which Josephus listed them, dates Theudas 15 years before the date Josephus gives[5]

*  Acts 11:28-9: Luke and Josephus both record famine during Claudius’ reign[6]

*  Acts 12:21-3: Luke describes Agrippa I’s death in a manner similar to Josephus, but with certain differences[7]

*  Acts 21:38: Luke describes ‘the Egyptian’ rebel leading sicarii into the wilderness but Josephus’s reference to sicarii in the wilderness is separate from his reference to ‘the Egyptian’[8]

*  Acts 25:13, 23; 26:30: Like Josephus, Luke implies that Agrippa II and Berenice are married, or consorts[9]

*  Acts 24:24-6: Like Josephus, Luke shows he is aware Drusilla (the wife of Felix), is a Jew

Scholarly Commentary

The claim is so insubstantial that most scholars consider it highly debatable at best,[10] rejecting it on a range of grounds and arguing Luke and Josephus used common traditions and historical sources. [11] [12] [13] [14] [15] [16] [17] [18] [19] [20] [21] [22] [23]

This consensus is even acknowledged by those who argue for Luke’s dependence on Josephus, or the other way around.[24]


[1] ‘This theory was maintained by F. C. Burkitt (The Gospel History and its Transmission, 1911, pp. 105–110), following the arguments of Krenkel’s Josephus und Lucas (1894).’, Guthrie, ‘New Testament Introduction’, p. 363 (4th rev. ed. 1996).

[2] Two recent examples are Richard Pervo’s ‘Dating Acts’ (2006), and ‘Acts: A Commentary’ in the series ‘Hermeneia: a Critical and Historical Commentary on the Bible’ (2008), and Steve Mason’s ‘Josephus and the New Testament’ (1992); Pervo’s is considered an academic argument worthy of response (though it has failed to convince most scholars), whereas Mason’s is rarely referred to in the relevant scholarly literature.

[3] ‘If Acts is dependent on Josephus for information, it cannot be earlier than 93. But such dependence is not proved and is highly unlikely.’, in Douglas & Tenney, ’New International Bible Dictionary’, p, 13 (1987).

[4] ‘A number of events to which allusion is possibly being made are discussed by J. Blinzler*, 32–37. These include: 1. the affair of the ensigns in Jos. Bel. 2:169–174; Ant. 18:55–59, but this took place in Caesarea in AD 26; 2. the tumults associated with the building of an aqueduct (Jos. Bel. 2:175–177; Ant. 18:60–62), but this incident involved the murder of Judaeans with cudgels outside the temple; 3. an attack on some Samaritans (Jos. Ant. 18:85–87), but this took place in AD 36; 4. the slaughter of about 3,000 Jews offering Passover sacrifices by Archelaus in 4 BC (Jos. Bel. 2:8–13; Ant. 17:213–218). This incident, however, took place some thirty years earlier and was committed by a different ruler; moreover, the murder of 3,000 men would not bear comparison with an accident to 18. It is wisest to conclude that the event is not attested from secular sources. This, however, is no argument against its historicity, since Josephus’ account of Pilate’s career is very incomplete (cf. Philo, Leg. 299-305). Pilate would have been in Jerusalem at Passover time, and the Galileans had a reputation for rebelliousness. The suggestion that Zealots were involved (O. Cullmann, The State in the NT, London, 1957, 14) lacks proof.’, Marshall, ‘The Gospel of Luke: A Commentary on the Greek Text’, New International Greek Testament Commentary, p. 553 (1978).

[5] ‘There are two problems: (1) Since Gamaliel was speaking well before AD 44 (the year in which Herod Agrippa I died, 12:20-23), a reference to the Theudas mentioned in Josephus would be anachronistic on his lips. (2). Gamaliel goes on to describe the rising of Judas after this; but the rising of Judas took place in AD 6 before the Theudas incident in Josephus. So, it is argued, Luke makes Gamaliel commit an anachronism and put the two stories in reverse chronological order. It has been argued that Luke was led to this error by misreading Josephus who goes on after the Theudas story to mention the sons of Judas and then to explain parenthetically who this Judas was and how he had led a revolt against Rome. But this supposition is highly unlikely, since Josephus’ works were not published till c. AD 93, and since Luke cannot possibly have got the details of his story (the 400 men) from him. No plausible explanation of Luke’s alleged error has been offered. There is, therefore, much to be said for the suggestions either that Josephus got his dating wrong or (more probably) that Gamaliel is referring to another, otherwise unknown Theudas. Since there were innumerable uprisings when Herod the Great died, and since ‘Josephus describes four men bearing the name of Simon within forty years and three that of Judas within ten years, all of whom were instigators of rebellion’ (cited by Knowling, p. 158), this suggestion should not be rejected out of hand.’, Marshall, ‘Acts: An Introduction And Commentary’, Tyndale New Testament Commentaries, volume 5, pp. 122-123 (1980).

[6] ‘Famines are mentioned in various parts of the empire during the time of Claudius. Josephus tells of a famine in Palestine during the governorship of Tiberius Alexander (46/48 C.E.):’, Conzelmann, Epp, & Matthews, ‘Acts of the Apostles: A Commentary on the Acts of the Apostles’, Hermeneia, p. 90 (1987).

[7]The details of Herod’s death are recorded slightly differently by Josephus, but the accounts are complementary. …Luke’s description of Herod as being eaten by worms is probably directly related to the abdominal pains referred to in Josephus’ account.’, Carson, ‘New Bible Commentary: 21st Century Edition’ (4th ed. 1994).

[8]According to Josephus (Bel. 2:261–263) there had been an Egyptian false prophet who had led 30,000 men to the Mount of Olives in order to take Jerusalem; he promised that they would see the walls of the city fall down. The governor, Felix, killed or captured his followers, while the prophet himself managed to escape. Clearly the tribune thought that this person had reappeared; the discrepancy between the number of his followers in Acts and in Josephus reflects the latter’s well-known tendency to exaggeration, and the tribune’s estimate will have been nearer the mark.’, Marshall, ‘Acts: An Introduction And Commentary’, Tyndale New Testament Commentaries, volume 5, p. 371 (1980).

[9] ‘There was gossip about the relationship between the brother and sister (Josephus Ant. 20.145; Juvenal Sat. 6.156–60). ‘,Conzelmann, Epp, & Matthews, ‘Acts of the Apostles: A Commentary on the Acts of the Apostles’, Hermeneia, p. 206 (1987).

[10] ‘The use of the LXX is not debatable, but the influence of Josephus and Paul has been and is subjected to considerable debate.’, Tyson, ‘Marcion and Luke-Acts: a defining struggle’, p. 14 (2006).

[11] ‘Arguments for the dependence of passages in Acts on Josephus (especially the reference to Theudas in Acts v. 37) are equally unconvincing. The fact is, as Schurer has said: “Either Luke had not read Josephus, or he had forgotten all about what he had read”‘, Geldenhuys, ‘Commentary on the Gospel of Luke’, p. 31 (1950).

[12] ‘But it is hardly logical to hold that Luke depends on Josephus and yet be obliged to admit that Luke shows wide divergence from him in relating events that are supposedly the same.’, Harrison, ‘Introduction to the New Testament’, p. 240 (1971).

[13] ‘The argument that Luke used the historian, Josephus (ad 93), was never fully convincing (HJ Cadbury, BC 11, 357). Today it is seldom pressed.’, Ellis, ‘The Gospel of Luke’, p. 55 (1977).

[14] ‘Sterling concludes that, while it is impossible to establish a literary dependence of Luke-Acts on the writings of Josephus, it is reasonable to affirm that both authors not only had access to similar historical traditions but also shared the same historiographical techniques and perspectives.’, Verheyden, ‘The Unity of Luke-Acts’, p. 678 (1990).

[15] ‘After examining the texts myself, I must conclude with the majority of scholars that it is impossible to establish the dependence of Luke-Acts on the Antiquitates. What is clear is that Luke-Acts and Josephos shared some common traditions about the recent history of Palestine.’, Sterling, ‘Historiography and Self-Definition: Josephos, Luke-Acts, and Apologetic Historiography’, Supplements to Novum Testamentum, pp. 365-366 (1992).

[16]It seems probable that Luke and Josephus wrote independently of one another; for each could certainly have had access to sources and information, which he then employed according to his own perspectives. A characteristic conglomerate of details, which in part agree, in part reflect great similarity, but also in part, appear dissimilar and to stem from different provenances, accords with this analysis.’, Schreckenberg & Schubert, ‘Jewish Historiography and Iconography in Early and Medieval Christian Literature’, Compendia Rerum Iudicarum Ad Novum Testamentum, volume 2, p. 51 (1992).

[17] ‘A. T. Robinson, Redating, p. 88, regards the Josephus line of approach as almost totally abandoned.’, Guthrie, ‘New Testament Introduction’, p. 364 (4th rev. ed. 1996).

[18] ‘From Krenkel’s remarks it can be seen that this proof can be offered only with very powerful mental contortions. See Hemer, Acts (n.37), 95: ‘the theory of Lukan dependence on Josephus has had in its day a certain vogue, and has been used as a major argument for the late dating of Luke-Acts’; cf. also Sterling, Historiography (n.37),365f. n.281.’, Hengel & Schwemer, Paul between Damascus and Antioch: the unknown years’, p. 325 (1997).

[19] ‘Nevertheless, direct literary dependence on Josephus by Luke is consistently dismissed for various reasons.’, Denova, ‘The Things Accomplished Among Us: prophetic tradition in the structural pattern of Luke-Acts’, p. 207 (1997).

[20] ‘The relationship between Luke and Josephus has produced an abundant literature, which has attempted to show the literary dependence of one on the other. I do not believe that any such dependence can be proved.’, Marguerat, ‘The First Christian Historian: writing the “Acts of the Apostles”‘, p. 79 (2002).

[21]Most scholars today deny any dependence one way or the other, and we think this judgment is correct.’, Heyler, ‘Exploring Jewish literature of the Second Temple Period: A Guide for New Testament Students’, p. 362  (2002).

[22] ‘When we consider both the differences and the agreement in many details of the information in the two accounts, [of the death of Herod Agrippa I] it is surely better to suppose the existence of a common source on which Luke and Josephus independently drew.’, Klauck & McNeil, ‘Magic and Paganism in Early Christianity: the world of the Acts of the Apostles’, p. 43 (2003).

[23] ‘Some attempt to argue a literary dependence on Josephus, and date Luke-Acts after 93CE. But, without a doubt, Luke’s theology is of an earlier type than Justin.’, Hear, ‘Simon Magus: the first gnostic?’, p. 71 (2003).

[24]Neither position has much of a following today, because of the significant differences between the two works in their accounts of the same events.’, Mason, ‘Josephus and the New Testament’, p. 185 (1992).

h1

Does the archaeological ‘Low Chronology’ disprove the Biblical narrative?

April 23, 2011

The Challenge

The ‘Low Chronology’ is a proposed redating of the Iron Age,[1] dating the reigns of David and Solomon to a time during which there is no archaeological evidence supporting them.[2]

The Objections

Proposed at least as early as the 1980s,[3]  the redating received almost no support,[4] and was resisted strongly by the archaeological consensus.[5]

Objections were raised by archaeologists including Dever (1997),[6] Mazar (1997, 1999),[7] Zarzeki-Peleg (1997),[8] Ben-Tor (1998), and Ben Ami (1998).[9]

Finkelstein responded, but criticism was renewed in 2000 by Na’aman[10] and Ben-Tor.[11] Over the next five years Finkelstein was virtually the only promoter of the theory. [12] [13] [14]  [15] [16] [17] [18]

The Evidence

Mazar and Dever note evidence agreeing with the Bible’s description of Jerusalem under David and Solomon.[19]  [20] Garfinkel likewise says evidence supports the description of the Israelite battles with the Philistines.[21]

Architecture at Khirbet Qeiyafa indicates David ruled an established state, as in the Biblical narrative.[22] Carbon 14 dated olive pits at the site have an age within the traditional date for the reign of David.[23]

Dead and Buried

Garfinkel believes the evidence from Khirbet Qeiyafa to be conclusive, [24] [25] and has declared ‘Low chronology is now officially dead and buried’.[26]


[1] ‘Proponents of the low chronology suggest that the end of the Iron Age I and the Iron Age IIA should be dated some eighty to one hundred years lower than the traditional chronology.’, Lehman, ‘The United Monarchy in the Countryside: Jerusalem, Judah and the Shephelah during the Tenth Century B.C.E’, in Vaughn & Killebrew (eds.), ‘Jerusalem in Bible and archaeology: The First Temple Period’, pp. 119-120 (2003; Iron Age I and Iron Age IIA are specific eras within the Iron Age.

[2]This suggested “Low Chronology” supposedly supports the replacement of this paradigm by a new one (in fact, similar to one presented earlier by David Jamieson Drake and others), according to which the kingdom of David and Solomon either did not exist or comprised at best a small local entity.’, Mazar, ‘The Search for David and Solomon: An Archaeological Perspective’, in Schmidt (ed.), ‘The Quest For the Historical Israel: debating archaeology and the history of Early Israel’, p. 119 (2007).

[3]“Revisionism” began on the archaeological front in the early 1980s, when several archaeologists o the Tel Aviv University set out to lower the conventional 10th century date of the distinctive four-entryway city gates and casement (or double) walls at Hazor, Megiddo, and Gezer to the early-mid-9th century BCE.’, Dever, ‘Biblical and Syro-Palestinian Archaeology’, in Perdue (ed.), ‘The Blackwell companion to the Hebrew Bible’, p. 137 (2001).

[4] ‘The Tel Aviv group’s idiosyncratic “low chronology,” however, was not accepted by the Jerusalem school, or by any European or American archaeologist (it still is not widely accepted, even by all Tel Aviv archaeologists).’, ibid., pp. 137-138.

[5] ‘The underlying premises of the Low Chronology were quickly challenged.’, Ortiz, ‘Deconstructing and Reconstructing the United Monarch’ , in Hoffmeier & Millard (eds.), ‘The Future of Biblical Archaeology: Reassessing Methodologies and Assumptions’, p. 128 (2004).

[6] ‘And I can tell you that not a single one of the other Israeli archaeologists agrees with this low chronology, except Israel Finkelstein.’ Dever, quoted in Shanks, ‘Face to Face: Biblical Minimalists Meet Their Challengers’, Biblical Archaeology Review (23.04), July/August 1997.

[7] ‘Mazar concluded that Finkelstein’s suggestion to push the date of the Philistine Monochrome pottery beyond the end of the Egyptian presence in Canaan is based on a debatable assumption (Tenet #2).’, ibid., p. 128.

[8] ‘A second article criticizing the Low Chronology was published by Anabel Zarzeki-Peleg.17 She also focused on the Iron Age stratigraphy of northern assemblages. Zarzeki-Peleg presented a ceramic typological study of three important northern sites (Megiddo, Jokneam, and Hazor) and concluded that the stratigraphical redating of the Low Chronology is not possible.’, ibid., p. 128.

[9] ‘The most significant studies, all opposed to Finkelstein’s “low chronology,” are those of Zarzeki-Peleg, 1997; Ben-Tor and Ben-Ami, 1998; and Mazar, 19991.’, Dever, ‘Biblical and Syro-Palestinian Archaeology’, in Perdue (ed.), ‘The Blackwell companion to the Hebrew Bible’, p. 202 (2001).

[10] ‘A second set of responses to Finkelstein’s Low Chronology came in a 2000 issue of BASOR.22 First, Nadav Na’aman challenged Finkelstein’s redating of the Philistine Monochrome pottery using Trojan Grey Ware from Lachish and Tel Miqne-Ekron.’, ibid, p. 129.

[11]‘A second article by Ben-Tor addressed Finkelstein’s redating of the northern sites, particularly Hazor.’, ibid., p. 129.

[12]‘In the meantime, his views are opposed by such leading archaeologists as Amihai Mazar of Hebrew University, excavator of Tel Rehov;* Amnon Ben-Tor of Hebrew University, excavator of Hazor;* Lawrence Stager of Harvard University, excavator of Ashkelon; and William Dever of the University of Arizona, excavator of Gezer. More to the point, Finkelstein’s low chronology has not been accepted even by his codirector at Megiddo, David Ussishkin. Ussishkin tells us that “on archaeological grounds it is quite possible (though not necessary) that some or all of [the structures in Stratum VA-IVB] originate in the 10th century B.C.E., during Solomon’s reign,” which is what the traditional chronology holds.’, Shanks, ‘Reviews:  Megiddo III—The 1992–1996 Seasons, Israel Finkelstein, David Ussishkin and Baruch Halpern, Editors’, Biblical Archaeology Review (6.06), November/December 2000.

[13] ‘What they do not tell the reader is that Finkelstein does not deny an Israelite state, but only down-dates its origins somewhat; and that his idiosyncratic “low chronology” is scarcely accepted by any other archaeologist.’, Dever, ‘What Did the Biblical Writers Know and When Did They Know it?’, p. 43 (2002).

[14] ‘It should not go unnoticed that not a single other ranking Syro-Palestinian archaeologist in the world has come out in print in support of Finkelstein’s ‘low chronology’.’, Dever, ‘Histories and Non-Histories of Ancient Israel: The Question of the United Monarchy’, in Day (ed.), ‘In Search of Pre-exilic Israel: proceedings of the Oxford Old Testament Seminar’, p. 73 (2003).

[15]‘The overwhelming consensus is, now more than ever, against Finkelstein’s low chronology, and therefore against his ‘new vision’ of ancient Israel.’, Dever, in Tel Aviv, volumes 30-31, p. 278 (2003).

[16]‘Demolishing Finkelstein’s supposed late date for the appearance of Philistine Bichrome pottery, based on an argument entirely from silence, leaves him without a leg to stand on for the remainder of his Iron I ‘low chronology’. While he continues to present it as fact, even claiming a growing consensus, there is not a shred of empirical (that is, stratigraphic) evidence to support this chronology.’, Dever, ‘Histories and Non-Histories of Ancient Israel: The Question of the United Monarchy’, in Day (ed.), ‘In Search of Pre-exilic Israel: proceedings of the Oxford Old Testament Seminar’, p. 73 (2003).

[17]‘Most senior archaeologists reject Finkelstein’s low chronology.’, Shanks, ‘Radiocarbon Dating: How to Find Your True Love’, Biblical Archaeology Review (31.01), January/February 2005; he cites ‘Amihai Mazar, Ephraim Stern, Amnon Ben-Tor, all of Hebrew University; Lawrence Stager of Harvard; William Dever and Seymour Gitin, the former and present directors of the Albright Institute in Jerusalem; and even Baruch Halpern, co-director with Finkelstein and David Ussishkin of the current excavation of Megiddo.’, but adds ‘But at this level of scholarship, you don’t simply count noses; you reason and argue! Recently, two brilliant younger archaeologists working at what is becoming a key site in the debate (Tel Dor on the Mediterranean coast) have parted company on this issue from their mentor, Hebrew University archaeologist Ephraim Stern, and now support Finkelstein’s low chronology.† Are the sands shifting?’.

[18] ‘Currently, Finkelstein is the only outspoken proponent of the Low Chronology.’, Ortiz, ‘Deconstructing and Reconstructing the United Monarch’ , in Hoffmeier & Millard (eds.), ‘The Future of Biblical Archaeology: Reassessing Methodologies and Assumptions’, p. 128 (2004).

[19] ‘Jerusalem of the tenth century B.C.E. is described by Finkelstein as a small and unimportant village. However, the “Stepped Stone Structure” in Area G in the City of David is a huge retaining wall that must have supported one of the largest buildings (perhaps the largest) of the 12th-10th centuries B.C.E. in the entire land of Israel. The pottery evidence indicates that it was founded during the Iron Age I (12th-11th centuries B.C.E.) and went out of use at some time after the tenth century. This fits the Biblical description of “The Citadel of Zion” (Metsudat Zion) as a Jebusite citadel captured by David and used as his stronghold (2 Samuel 5:7). In addition, Iron IIA pottery was found in almost every excavation area in the City of David. Jerusalem may not have been an enormous city during that time, but it definitely was much more than merely a small village, as Finkelstein contends. Outside of Jerusalem, monumental structures at Hazor, Megiddo and Gezer can, in my opinion, be dated to the tenth century B.C.E. Thus Yigael Yadin was probably correct in suggesting that these should be associated with Solomon’s building projects mentioned in 1 Kings 9:15.’, Mazar, ‘Does Amihai Mazar Agree with Finkelstein’s “Low Chronology”?’, Biblical Archaeology Review (29.02), March/April 2003.

[20] ‘If the biblical Solomon had not constructed the Gezer gate and city walls, then we would have to invent a similar king by another name.’, Dever, ‘What Did the Biblical Writers Know and When Did They Know it?’, p. 133 (2002).

[21]The geopolitical circumstances in the Elah Valley during the late 11th–early 10th centuries are quite clear. The mighty Philistine city state of Gath, ca. 30 hectares in area, was located only 12 km downstream from Khirbet Qeiyafa. This was a hostile border area, where the Kingdoms of Gath and Jerusalem had constant millenary conflicts. The story of David and Goliath is just one of many such “warrior tales” listed in 2 Sam 21:15–22 and 1 Ch 11:11–27. Even if many of these traditions are folkloristic in character, their chronology and geography bear historical memories. As by the end of the 9th century BCE Gath disappeared as a political power, these traditions must have been created at an earlier time.’, Garfinkel, ‘Khirbet Qeiyafa: Sha’arayim’, Journal of Hebrew Scriptures (8.22.6). 2008.

[22]Khirbet Qeiyafa is surrounded by a massive casemate city wall, 700 m long and 4 m wide. It is constructed of megalithic stones, quite often reaching a weight of 4–5 tons apiece, and in the eastern gate, even ca. 10 tons each. Our calculation suggests that 200,000 tons of stone were required for the construction of these fortifications. A four-chambered gate, its upper part constructed of ashlars, was located and excavated in the western part of the city. It is clearly a fortified town rather than a rural settlement.’, Garfinkel, ‘Khirbet Qeiyafa: Sha’arayim’, Journal of Hebrew Scriptures (8.22.5). 2008.

[23] ‘As Khirbet Qeiyafa is an Iron Age IIA site, we are left with a dating post-1000 BCE, that is, 1000–975 BCE (59.6%) or 1000–969 BCE (77.8%). These dates fit the time of King David (ca. 1000–965 BCE) and are too early for King Solomon (ca. 965–930 BCE).’, ibid., p. 3.

[24] ‘The four new C14 results from Khirbet Qeiyafa clearly indicate that the “low chronology” and the “ultra-low chronology” are unacceptable.’, ibid., p. 4-5.

[25] ‘The biblical text, the single-phase city at Khirbet Qeiyafa, and the radiometric dates each stands alone as significant evidence clearly indicating that the biblical tradition does bear authentic geographical memories from the 10th century BCE Elah Valley. There is no ground for the assumption that these traditions were fabricated in the late 7th century BCE or in the Hellenistic period.’, ibid., pp. 5-6.

[26] Garfinkel & Ganor, ‘Khirbet Qeiyafa: An Early Iron IIa Fortified City in Judah’, presentation to the American Schools of Oriental Research, slide 24 (2010).